83
ĐẠI HỌC QUỐC GIA HÀ NỘI TRƯỜNG ĐẠI HỌC KHOA HỌC TỰ NHIÊN ————————– VŨ THỊ HIỀN SÁU PHƯƠNG PHÁP GIẢI CÁC BÀI TOÁN PHỔ THÔNG LUẬN VĂN THẠC SĨ KHOA HỌC Hà Nội - 2015

SÁU PHƯƠNG PHÁP GIẢI CÁC BÀI TOÁN PHỔ THÔNG

  • Upload
    vanliem

  • View
    227

  • Download
    1

Embed Size (px)

Citation preview

Page 1: SÁU PHƯƠNG PHÁP GIẢI CÁC BÀI TOÁN PHỔ THÔNG

ĐẠI HỌC QUỐC GIA HÀ NỘITRƯỜNG ĐẠI HỌC KHOA HỌC TỰ NHIÊN

————————–

VŨ THỊ HIỀN

SÁU PHƯƠNG PHÁP GIẢI CÁCBÀI TOÁN PHỔ THÔNG

LUẬN VĂN THẠC SĨ KHOA HỌC

Hà Nội - 2015

Page 2: SÁU PHƯƠNG PHÁP GIẢI CÁC BÀI TOÁN PHỔ THÔNG

ĐẠI HỌC QUỐC GIA HÀ NỘITRƯỜNG ĐẠI HỌC KHOA HỌC TỰ NHIÊN

————————–

VŨ THỊ HIỀN

SÁU PHƯƠNG PHÁP GIẢI CÁCBÀI TOÁN PHỔ THÔNG

Chuyên ngành: Phương pháp toán sơ cấp

Mã số : 60460113

LUẬN VĂN THẠC SĨ KHOA HỌC

Người hướng dẫn khoa học: GS.TS. Đặng Huy Ruận

Hà Nội - 2015

Page 3: SÁU PHƯƠNG PHÁP GIẢI CÁC BÀI TOÁN PHỔ THÔNG

Mục lục

Mở đầu 1

1 Phương pháp quy nạp 21.1 Nguyên lý quy nạp . . . . . . . . . . . . . . . . . . . . . . . . . . . 21.2 Phương pháp chứng minh bằng quy nạp . . . . . . . . . . . . . . . 2

1.2.1 Cơ sở quy nạp . . . . . . . . . . . . . . . . . . . . . . . . . . 31.2.2 Quy nạp . . . . . . . . . . . . . . . . . . . . . . . . . . . . . 31.2.3 Vận dụng phương pháp quy nạp để giải một số bài toán . 4

2 Phương pháp chứng minh phản chứng 172.1 Cơ sở lý thuyết . . . . . . . . . . . . . . . . . . . . . . . . . . . . . 172.2 Nội dung của phương pháp phản chứng . . . . . . . . . . . . . . . 182.3 Trình bày lời giải của phương pháp phản chứng . . . . . . . . . . . 192.4 Một số ví dụ minh họa . . . . . . . . . . . . . . . . . . . . . . . . . 19

3 Phương pháp suy luận trực tiếp 283.1 Vài nét về phương pháp suy luận trực tiếp . . . . . . . . . . . . . . 283.2 Các ví dụ về vận dụng phương pháp suy luận trực tiếp . . . . . . 29

4 Phương pháp đồ thị 354.1 Một số khái niệm và kết quả cơ bản của lí thuyết đồ thị . . . . . . 354.2 Phương pháp đồ thị . . . . . . . . . . . . . . . . . . . . . . . . . . . 36

4.2.1 Xây dựng đồ thị mô tả các quan hệ . . . . . . . . . . . . . 374.2.2 Dựa vào các kết quả của lý thuyết đồ thị hoặc lý luận trực

tiếp suy ra đáp án của bài toán D . . . . . . . . . . . . . . 374.3 Một số ví dụ . . . . . . . . . . . . . . . . . . . . . . . . . . . . . . . 37

5 Phương pháp bảng 535.1 Giới thiệu về phương pháp bảng . . . . . . . . . . . . . . . . . . . . 53

i

Page 4: SÁU PHƯƠNG PHÁP GIẢI CÁC BÀI TOÁN PHỔ THÔNG

MỤC LỤC

5.2 Một số ví dụ minh họa . . . . . . . . . . . . . . . . . . . . . . . . . 53

6 Phương pháp sơ đồ 676.1 Các bước thực hiện phương pháp sơ đồ . . . . . . . . . . . . . . . 67

6.1.1 Thiết lập sơ đồ . . . . . . . . . . . . . . . . . . . . . . . . . 676.1.2 Dựa vào cấu trúc của sơ đồ mô tả quan hệ và điều kiện

đã cho trong bài toán mà suy ra đáp án . . . . . . . . . . . 676.2 Một số ví dụ . . . . . . . . . . . . . . . . . . . . . . . . . . . . . . . 67Kết luận . . . . . . . . . . . . . . . . . . . . . . . . . . . . . . . . . . . . 77Tài liệu tham khảo . . . . . . . . . . . . . . . . . . . . . . . . . . . . . 79

ii

Page 5: SÁU PHƯƠNG PHÁP GIẢI CÁC BÀI TOÁN PHỔ THÔNG

Mở đầu

Toán phổ thông chẳng những nhiều về số lượng, còn phong phú về chủngloại.

Mỗi chủng loại đòi hỏi một phương pháp giải thích hợp. Bởi vậy có nhiềuphương pháp giải toán phổ thông.

Với khối lượng có hạn, luận văn chỉ xin phép trình bày sáu trong nhữngphương pháp thường dùng nhất.

Luận văn gồm phần mở đầu và sáu chương:Chương I trình bày về phương pháp quy nạp,Chương II trình bày về phương pháp phản chứng,Chương III trình bày về phương pháp suy luận trực tiếp,Chương IV trình bày về phương pháp đồ thị,Chương V trình bày về phương pháp bảng,Chương V I trình bày về phương pháp sơ đồ.Mỗi phương pháp đều có phần tóm tắt cơ sở lý thuyết và phần vận dụng

phương pháp để giải bài tập.Luận văn được hoàn thành dưới sự hướng dẫn tận tình của thầy giáo GS. TS

Đặng Huy Ruận. Em xin bày tỏ lòng kính trọng và biết ơn sâu sắc đến Thầy.Em xin trân trọng cảm ơn ban lãnh đạo khoa Toán - Cơ - Tin học, khoa SauĐại học, Trường Đại học Khoa học tự nhiên, Đại học Quốc Gia Hà Nội, cácThầy, Cô giáo đã trang bị kiến thức, tạo điều kiện cho chúng em trong thời gianhọc tập tại đây. Tôi cũng xin gửi lời cảm ơn đến các Đồng nghiệp tại trườngPhổ Thông Hồng Đức - Hà Nội, những người đã động viên giúp đỡ tôi rất nhiềutrong quá trình hoàn thành luận văn này.

Luận văn khó tránh khỏi hạn chế và sơ xuất. Rất mong được sự chỉ bảo củaQuý thầy cô và Quý bạn đọc để luận văn được hoàn thiện hơn.

Tôi xin chân thành cảm ơn!

1

Page 6: SÁU PHƯƠNG PHÁP GIẢI CÁC BÀI TOÁN PHỔ THÔNG

Chương 1

Phương pháp quy nạp

Phương pháp quy nạp có vai trò vô cùng quan trọng trong toán học, khoahọc và cuộc sống. Đối với nhiều bài toán trong chương trình toán phổ thông lànhững bài toán logic, tức những bài toán không mẫu mực phương pháp quy nạpcho ta nhiều cách giải hữu hiệu.

Suy diễn là quá trình từ "tính chất" của tập thể suy ra tính chất của cá thể,nên luôn luôn đúng, còn quá trình ngược lại, tức quá trình quy nạp: đi từ "tínhchất" của một số các thể suy ra "tính chất" của tập thể thì không phải lúc nàocũng đúng, mà quá trình này chỉ đúng khi nó thỏa mãn một số điều kiện nàođó, tức thỏa mãn nguyên lý quy nạp.

1.1 Nguyên lý quy nạp

Nếu khẳng định S(n) thỏa mãn hai điều kiện sau:a) Đúng với n = k0 (số tự nhiên nhỏ nhất mà S(n) xác định).b) Từ tính đúng đắn của S(n) đến n = t (hoặc đối với mọi giá trị của n (k0 ≤n ≤ t)) (t ≥ k0), ta cần chứng minh tính đúng đắn của S(n) đối với n = t+1, thìkhiØS(n) đúng với mọi n ≥ k0.

1.2 Phương pháp chứng minh bằng quy nạp

Giả sử khẳng định S(n) xác định với mọi n ≥ t0. Để chứng minh S(n) đúng∀n ≥ t0 bằng quy nạp ta cần thực hiện theo hai bước sau:

2

Page 7: SÁU PHƯƠNG PHÁP GIẢI CÁC BÀI TOÁN PHỔ THÔNG

Chương 1. Phương pháp quy nạp

1.2.1 Cơ sở quy nạp

Thực hiện bước này tức là ta thử xem sự đúng đắn của S(n) với n = t0 nghĩalà xét S(t0) có đúng hay không?

1.2.2 Quy nạp

Giả sử khẳng định S(n) đã đúng đến n = t (hoặc đối với mọi n (t0 ≤ n ≤ t))

(t ≥ t0). Trên cơ sở giả thiết này ta chứng minh tính đúng đắn của S(n) đối vớin = t + 1, tức S(t+ 1) đúng.

Nếu cả ba bước trên thỏa mãn, thì theo nguyên lý quy nạp S(n) đúng với∀n ≥ t0.Chú ý: Trong quá trình quy nạp nếu không thực hiện đầy đủ cả ba bước: Cơsở quy nạp, giả thiết quy nạp và chứng minh quy nạp, thì có thể dẫn đến kếtquả sai lầm, chẳng hạn:

- Do bỏ bước cơ sở quy nạp, ta đưa ra kết luận không đúng: Mọi số tự nhiênđều bằng nhau! Bằng cách quy nạp như sau: Giả sử các số tự nhiên không vượtquá k + 1 đã bằng nhau. Khi đó ta có

k = k + 1

Thêm vào mỗi vế của đẳng thức trên một đơn vị ta có

k + 1 = k + 1 + 1 = k + 2

Cứ như vậy suy ra mọi số tự nhiên không nhỏ hơn k đều bằng nhau. Kết hợpvới giả thiết quy nạp: Mọi số tự nhiên không vượt quá k đều bằng nhau, đi đếnkết luận sai lầm: Tất cả các số tự nhiên đều bằng nhau!

- Do bỏ qua khâu quy nạp nên nhà toán học Pháp P.Fermat (1601-1665) đãcho rằng các số dạng 22

n

+ 1 đều là số nguyên tố.P.Fermat xét 5 số đầu tiên:

Với n = 0 cho 220

+ 1 = 21 + 1 = 3 là số nguyên tố.

n = 1 cho 221

+ 1 = 22 + 1 = 5 là số nguyên tố.

n = 2 cho 222

+ 1 = 24 + 1 = 17 là số nguyên tố.

n = 3 cho 223

+ 1 = 28 + 1 = 257 là số nguyên tố.

n = 4 cho 224

+ 1 = 216 + 1 = 65537 là số nguyên tố.

3

Page 8: SÁU PHƯƠNG PHÁP GIẢI CÁC BÀI TOÁN PHỔ THÔNG

Chương 1. Phương pháp quy nạp

Nhưng vào thế kỷ 18 Euler đã phát hiện với n = 5 khẳng định trên khôngđúng, bởi vì:

225

+ 1 = 4294967297 = 641× 6700417

là hợp số.

1.2.3 Vận dụng phương pháp quy nạp để giải một số bài toán

Phương pháp quy nạp được sử dụng trong tính toán, trong chứng minh vàtrong suy luận dưới nhiều dạng khác nhau, nhưng trong phần này chỉ trình bàyviệc vận dụng phương pháp quy nạp để giải các bài toán logic, tức các bài toán"không mẫu mực".

Ví dụ 1.2.1. Chứng minh rằng: Nếu trong túi có một số tiền nguyên (nghìn)không ít hơn 8000đ, thì luôn luôn có thể mua vé sổ số loại 5000đ và 3000đ.

Lời giải: Ta sẽ giải quyết bài toán này bằng phương pháp quy nạp.1) Cơ sở quy nạp. Nếu trong túi có số tiền ít nhất, tức 8000đ, thì ta mua một

vé sổ số loại 5000đ và một vé sổ số loại 3000đ. Khi đó

1× 5000đ+ 1× 3000đ = 8000đ

và ta đã tiêu được hết số tiền có trong túi.2) Quy nạp. Giả sử với k(k ≥ 8000) nghìn đồng ta đã tiêu hết bằng cách mua

các vé sổ số loại 5000đ và 3000đ. Nếu có thêm 1000đ nữa ta cũng có thể muađược bằng cách sau đây:

a) Nếu trong các vé sổ số đã mua có ít nhất ba vé loại 3000đ, thì ta trả lạiba vé loại 3000đ, đưa thêm 1000đ và lấy về hai vé loại 5000đ. Khi đó

3× 3000đ+ 1000đ = 2× 5000đ.

b) Nếu trong các vé sổ số đã mua có không quá hai vé loại 3000đ, thì phảicó ít nhất một vé loại 5000đ. Bởi vì trong túi không ít hơn 8000đ, mà đã tiêuhết. Khi đó đem trả lại một vé loại 5000đ, đưa thêm 1000đ và lấy về hai vé loại3000đ, ta có

1× 5000đ+ 1000đ = 2× 3000đ

Như vậy trong mọi trường hợp từ kết quả tiêu k nghìn đầu tiên đã suy rađược cách tiêu nghìn thứ k + 1, nên bài toán đã được giải quyết xong.

4

Page 9: SÁU PHƯƠNG PHÁP GIẢI CÁC BÀI TOÁN PHỔ THÔNG

Chương 1. Phương pháp quy nạp

Ví dụ 1.2.2. Em An cầm một tờ giấy và lấy kéo cắt thành 7 mảnh. Sau đó nhặtmột trong những mảnh giấy đã cắt và lại cắt thành 7 mảnh. Và em An cứ tiếptục cắt giấy như vậy. Sau một hồi em An thu tất cả các mẩu giấy đã cắt ra vàđếm được 122 mảnh. Liệu em An đếm đúng hay sai?

Lời giải:1) Mỗi lần cắt mảnh giấy thành 7 mảnh, tức là đã tạo ra thêm 6 mảnh giấy,

nên công thức tính số mảnh giấy sau n bước thực hiện một mảnh giấy thành 7

mảnh có dạng: S(n) = 6n + 1.2) Tính đúng đắn của công thức S(n) được khẳng định bằng quy nạp theo n.10) Cơ sở quy nạp. Với n = 1, em An cắt mảnh giấy có trong tay thành 7

mảnh, nên cóS(1) = 6.1 + 1 = 6 + 1 = 7

20) Quy nạp. Giả sử sau k bước em An đã nhận được số mảnh giấy là

S(k) = 6k + 1

Sang bước k + 1 em An lấy một trong những mảnh giấy nhận được trong k

bước trước và cắt thành 7 mảnh, tức em An đã lấy đi một trong S(k) mảnh vàthay vào đó 7 mảnh được cắt ra nên

S(k + 1) = S(k)− 1 + 7 = 6k + 1− 1 + 7 = 6k + 7

= 6k + 6 + 1 = 6(k + 1) + 1

Vậy số mảnh giấy em An nhận được sau n bước cắt giấy là S(n).3) Do S(n) = 6n + 1 ≡ 1 (mod 6), nhưng 122 = 6.20 + 2 ≡ 2 (mod 6), nên em

An đếm không đúng. �

Ví dụ 1.2.3. (Chứng minh tính chất bằng quy nạp).Cho x + 1

x, x 6= 0 là một số nguyên. Chứng minh rằng với mọi số nguyên dương

n, sốT (n, x) = xn +

1

xn

cũng là số nguyên.

Lời giải: Khẳng định được chứng minh bằng quy nạp.1) Cơ sở quy nạp. Với n = 1 có T (1, x) = x+ 1

xlà số nguyên, theo giả thiết.

2) Quy nạp. Giả sử khẳng định đúng với mọi số nguyên k(n ≥ k ≥ 1) nghĩalà

T (k, x) = xk +1

xk

5

Page 10: SÁU PHƯƠNG PHÁP GIẢI CÁC BÀI TOÁN PHỔ THÔNG

Chương 1. Phương pháp quy nạp

là số nguyên.Với n = k + 1 số

T (k + 1, x) = xk+1 +1

xk+1=

=(

x+1

x

)(

xk +1

xk

)

−(

xk−1 +1

xk−1

)

theo giả thiết quy nạp, các số x + 1x, xk−1 + 1

xk−1 và xk + 1xk đều nguyên, nên

T (k + 1, x) là số nguyên và khẳng định đúng với mọi số nguyên dương n. �

Ví dụ 1.2.4. Chứng minh rằng mọi số nguyên lớn hơn 1 đều có thể viết dướidạng tích của các thừa số nguyên tố.

Lời giải: Ta chứng minh khẳng định bằng quy nạp theo n.1. Cơ sở quy nạp. Với n = 2, ta có 2 = 2,

Với n = 3, ta có 3 = 3, n = 4, ta có 4 = 2× 2

Vậy khẳng định đúng với n = 2, 3, 4.2. Quy nạp. Giả sử với mọi số nguyên n đều phân tích được thành tích của

các thừa số nguyên tố. Ta chứng minh n+1 cũng phân tích được thành tích củacác thừa số nguyên tố.

Thật vậy

• Nếu n+ 1 là số nguyên tố thì nó bằng tích của chính n + 1.

• Nếu n+ 1 là hợp số thì n+ 1 = a.b với 2 ≤ a, b < n.

Theo giả thiết quy nạp, thì a, b đều phân tích được thành tích các thừa sốnguyên tố.

Suy ra, n+ 1 cũng phân tích được thành tích các thừa số nguyên tố.Theo nguyên lý quy nạp, mọi số nguyên n > 1 đều phân tích được thành tích

các thừa số nguyên tố. �

Ví dụ 1.2.5. (Chứng minh tính chia hết bằng quy nạp). Chứng minh rằng với

mọi số nguyên dương n số 23n

+1 chia hết cho 3n+1[

(

23n

+ 1) ...3n+1

]

và số 23n

+1

không chia hết cho 3n+2[

(

23n

+ 1)

6 ...3n+2]

.

Lời giải: Bài toán được giải quyết bằng quy nạp. Kí hiệu 23n

+ 1 = An.1) Cơ sở quy nạp.

Với n = 1 ta có A1 = 231

+ 1 = 23 + 1 = 8 + 1 = 9, nên A1...32 và A1 6

...33.Với n = 2 ta có A2 = 23

2

+ 1 = 513, nên A2...33 và A2 6

...34.

6

Page 11: SÁU PHƯƠNG PHÁP GIẢI CÁC BÀI TOÁN PHỔ THÔNG

Chương 1. Phương pháp quy nạp

2) Quy nạp. Giả sử khẳng định đã đúng với n = k ≥ 2, nghĩa là Ak

...3k+1 vàAk 6 ...3k+2.

Vì Ak

...3k+1, nên

∃M ∈ N (Ak = M.3k+1 và M 6 ...3) (1.1)

Xét n = k + 1

Ak+1 = 23k+1

+ 1 = 23k.3 + 1 =

(

23k

)3

+ 1 =(

23k

+ 1)

(

(

23k

)2

− 23k

+ 1

)

= 3k+1.M.

[

(

23k

+ 1)2

− 3.23k

]

= 3k+1.M.[

(

3k+1.M)2 − 3.23

k

]

= 3k+1.M.[

32k+2.M2 − 3.23k

]

= 3k+2.M.[

32k+1.M2 − 23k

]

10) Khi đó Ak+1...3k+2, nên với mọi số nguyên dương n đều có 23

n

+ 1...3n+1.

20) Ak+1 6...3k+3.

a) Vì M 6 ...3 (theo (1.1)), nên3k+2.M 6 ...3k+3 (1.2)

b) Do k ≥ 2, nên ∃t ∈ N (k = t+ 2) và 32k+1.M2 = 3k+3+t.M2.

Bởi vậy32k+1.M2...3k+3 (1.3)

Giả sử 23k ...3k+3. Khi đó 23

k ...9, nhưng 23k

=(

23)k

= 8k = ±1 (mod 9), nên

23k 6 ...3k+3 (1.4)

Từ các quan hệ (1.3) và (1.4) ta suy ra(

32k+1.M2 − 23k

)

6 ...3k+3 (1.5)

Từ (1.2) và (1.5) suy ra: Ak+1 6...3k+3, nên với mọi số nguyên dương n số An không

chia hết cho 3n+2. �

Ví dụ 1.2.6. (Chứng minh bất đẳng thức bằng quy nạp)Cho n(n ≥ 1) số dương x1, x2, · · · , xn thỏa mãn điều kiện

x1.x2. · · ·xn−1.xn = 1

Chứng minh rằngx1 + x2 + · · ·+ xn−1 + xn ≥ n

và dấu đẳng thức xảy ra khi và chỉ khi x1 = x2 = · · · = xn

7

Page 12: SÁU PHƯƠNG PHÁP GIẢI CÁC BÀI TOÁN PHỔ THÔNG

Chương 1. Phương pháp quy nạp

Lời giải: Bài toán được giải quyết bằng quy nạp.1) Cơ sở quy nạp. Với n = 1 ta chỉ có số x1. Vì x1 = 1, nên x1 thỏa mãn bất

đẳng thức x1 ≥ 1.2) Quy nạp. Giả sử khẳng định đã đúng với k số dương tùy ý có tích bằng 1.

Xét k+1 số dương tùy ý x1, x2, · · · , xk, xk+1 với x1.x2. · · ·xk.xk+1 = 1. Có hai khảnăng đặt ra:a) Nếu x1 = x2 = · · · = xk = xk+1, thì xi = 1 (1 ≤ i ≤ k + 1). Khi đó

x1 + x2 + · · ·+ xk + xk+1 = k + 1

và khẳng định được chứng minh.b) Nếu k + 1 số được xét x1, x2, · · · , xk, xk+1 không đồng thời bằng nhau, thì dotích của chúng bằng 1 và các số này đều dương, phải có ít nhất một số lớn hơn1 và ít nhất một số nhỏ hơn 1. Không giảm tính tổng quát, giả sử xk < 1 vàxk+1 > 1. Khi đó

1− xk > 0, xk+1 − 1 > 0 nên (1− xk) (xk+1 − 1) > 0

Bởi vậyxk + xk+1 > xk.xk+1 + 1 (1.6)

Từ đẳng thức: x1.x2. · · ·xk−1 (xkxk+1) = x1.x2. · · · .xk−1xk.xk+1 = 1 suy ra k sốdương x1, x2, · · · , xk−1, (xkxk+1) có tích bằng 1, nên theo giả thiết quy nạp, có

x1 + x2 + · · ·+ xkxk+1 ≥ k

Cộng cả hai vế của bất đẳng thức trên với 1 ta có bất đẳng thức

x1 + x2 + · · ·+ xkxk+1 + 1 ≥ k + 1 (1.7)

Từ bất đẳng thức (1.6) và (1.7) ta có

x1 + x2 + · · ·+ xk + xk+1 ≥ x1 + x2 + · · ·+ xkxk+1 + 1 ≥ k + 1

Khẳng định đã được chứng minh.Với n số dương tùy ý x1 = x2 = · · · = xn và x1.x2. · · ·xn = 1 suy ra xi = 1(1 ≤ i ≤n), nên x1 + x2 + · · ·+ xn = n. �

Ví dụ 1.2.7. (Tìm chữ số tận cùng bằng quy nạp)Với mọi số nguyên dương k ≥ 2 hãy tìm chữ số tận cùng của số Ak = 22

k

+ 1.

8

Page 13: SÁU PHƯƠNG PHÁP GIẢI CÁC BÀI TOÁN PHỔ THÔNG

Chương 1. Phương pháp quy nạp

Lời giải: Bài toán được giải quyết bằng quy nạp.1) Cơ sở quy nạp. Với k = 2 số A2 = 22

2

+ 1 = 24 + 1 = 17

2) Quy nạp. Giả sử với k = n ≥ 2 số An đã có tận cùng là 7.Xét số An+1 = 22

n+1

+ 1.Do An tận cùng số 7, nên tồn tại số nguyên dương m, để An = 10m+ 7. Từ đó

An − 1 = 22n

= 10m+ 6.

An+1 = 22n+1

+ 1 = 22n.2

+ 1 =(

22n)2

+ 1

= (10m+ 6)2 + 1 = 100m2 + 120m+ 36 + 1

= 10(

10m2 + 12m)

+ 37 = 10(

10m2 + 12m+ 3)

+ 7

nên An+1 tận cùng bằng chữ số 7.Vậy với mọi số k ≥ 2, số Ak tận cùng bằng chữ số 7.

Ví dụ 1.2.8. Tính tổng sau

S(n) = 12 + 32 + · · ·+ (2n+ 1)2 ,

trong đó n là một số tự nhiên.

Lời giải: Ta sẽ đi dự đoán công thức tổng S(n). Ta thấy S(n) là tổng của cáclũy thừa bậc hai của các số 1, 3, · · · , (2n+ 1), nên ta dự đoán S(n) phải là mộtđa thức của n có bậc không nhỏ hơn ba. Giả sử S(n) = an3 + bn2 + cn + d. VìS(0) = 1 nên d = 1. Lần lượt thay n = 1, n = 2, n = 3, ta được hệ

a+ b+ c = 9

8a + 4b+ 2c = 34

27a+ 9b+ 3c = 83

Giải hệ này ta thu được a = 43 , b = 4, c = 11

3 . Khi đó,

S(n) =4

3n3 + 4n2 +

11

3n + 1 =

(n+ 1) (2n+ 1) (2n+ 3)

3(1.8)

Ta sẽ chứng minh công thức (1.8) bằng quy nạp theo n.1) Cơ sở quy nạp. Với n = 0, 1, 2, 3, 4, 5, ta dễ dàng kiểm tra được (1.8) đúng.2) Quy nạp. Giả sử (1.8) đúng với n = k, ta chứng minh (1.8) đúng với

n = k + 1.Thật vậy

S(k + 1) = 12 + 32 + · · ·+ (2k + 1)2 + (2k + 3)2

= S(k) + (2k + 3)2 .

9

Page 14: SÁU PHƯƠNG PHÁP GIẢI CÁC BÀI TOÁN PHỔ THÔNG

Chương 1. Phương pháp quy nạp

Theo giả thiết quy nạp thì S(k) = (k+1)(2k+1)(2k+3)3 .

Suy ra

S(k + 1) =(k + 1) (2k + 1) (2k + 3)

3+ (2k + 3)2

=(k + 2) (2k + 3) (2k + 5)

3

Vậy công thức (1.8) đúng với n = k+1, do đó theo nguyên lý quy nạp (1.8) đúngvới mọi n.

Ví dụ 1.2.9. Chứng minh rằng A(n) = 7n + 3n− 1 chia hết cho 9 với mọi số tựnhiên n.

Lời giải: Đặt A(n) = 7n + 3n− 1. Ta sẽ chứng minh bài toán bằng quy nạp.1) Cơ sở quy nạp. Với n = 0, ta có A(0) = 0 chia hết cho 9.2) Quy nạp. Giả sử A(k) chia hết cho 9 với mọi k ∈ N. Ta sẽ chứng minh

A(k + 1) cũng chia hết cho 9.Thật vậy, ta có

A(k + 1) = 7k+1 + 3 (k + 1)− 1

= 7A(k)− 9. (2k − 1) .(1.9)

Theo giả thiết quy nạp thì A(k) chia hết cho 9. Do đó từ (1.9) ta suy ra A(k+1)

cũng chia hết cho 9.Vậy A(n) chia hết cho 9 với mọi số tự nhiên n. �

Ví dụ 1.2.10. Gọi S(n) là tổng tất cả các ước số lẻ lớn nhất của các số tự nhiêntừ 1 đến 2n. Chứng minh rằng 3S(n) = 4n + 2.

Lời giải: Các số tự nhiên từ 1 đến 2n bao gồm các số lẻ từ 1 đến 2n và gấp đôicủa các số tự nhiên từ 1 đến 2n−1. Từ đó ta có

S(n) = S(n− 1) + 1 + 3 + · · ·+ (2n − 1)

hayS(n) = S(n− 1) + 4n−1 (1.10)

Ta sẽ chứng minh bài toán đã cho bằng quy nạp theo n.1) Cơ sở quy nạp. Với n = 1, ta có 3S(1) = 3(1 + 1) = 6 = 41 + 2, khẳng định

đúng với n = 1

10

Page 15: SÁU PHƯƠNG PHÁP GIẢI CÁC BÀI TOÁN PHỔ THÔNG

Chương 1. Phương pháp quy nạp

2) Quy nạp. Giả sử khẳng định đúng với n = k, tức là

3S(k) = 4k + 2 (1.11)

Ta cần chỉ ra khẳng định đúng với n = k + 1 hay 3S(k + 1) = 4k+1 + 2.Thật vậy, theo (1.10) ta có

S(k + 1) = S(k) + 4k

⇔ 3S(k + 1) = 3S(k) + 3.4k

= 4k + 2 + 3.4k

= 4k+1 + 2.

(1.12)

theo giả thiết quy nạp. Vậy khẳng định đã cho đúng với mọi n ∈ N∗. �

Ví dụ 1.2.11. Tìm bậc cao nhất k của 2007 sao cho 2007k là ước của số:

200820072006

+ 200620072008

.

Lời giải: Ta chứng minh hai bổ đề sau:

Bổ đề 1.2.1. Với số tự nhiên lẻ a ≥ 3 và với mọi n nguyên dương không chiahết cho a, ta có:

(1 + a)an

= 1 + Snan+1,

trong đó Sn là số nguyên dương và không chia hết cho a.

Bổ đề 1.2.2. Với số tự nhiên lẻ b ≥ 3 và với mọi số n nguyên dương, ta có:

(b− 1)bn

= −1 + tnbn+1,

trong đó tn là số nguyên không chia hết cho b.

+, Chứng minh bổ đề 1: Ta sẽ chứng minh bổ đề 1 bằng quy nạp.1. Cơ sơ quy nạp. Với n = 1, ta có

(1 + a)a = 1 + C1aa+ C2

aa2 + . . .+ Ca

aaa =

= 1 + a2(

1 + C2a + C3

aa+ . . .+ aa−2)

= 1 + S1a2.

Vì a lẻ nên C2a = a!

2!(a−2)! =a(a−1)

2

...a.Do vậy, S1 = 1 + C2

a + C3aa+ . . .+ aa−2 không chia hết cho a.

11

Page 16: SÁU PHƯƠNG PHÁP GIẢI CÁC BÀI TOÁN PHỔ THÔNG

Chương 1. Phương pháp quy nạp

2. Quy nạp. Giả sử khẳng định trên đúng với n = k, tức là

(1 + a)ak

= 1 + Skak+1,

trong đó Sk là số nguyên dương và không chia hết cho a.Ta có

(1 + a)ak+1

= (1 + a)ak.a =

(

1 + Skak+1

)a

=(

1 + C1aSka

k+1 + C2a

(

Skak+1

)2+ . . .+ Ca

a

(

Skak+1

)a)

= 1 + ak+2[

Sk + C2aS

2k + . . .+ Sa

kaak+a−k−2

]

= 1 + Sk+1ak+2.

Vì Sk không chia hết cho a suy ra Sk+1 không chia hết cho a.Vậy bổ đề được chứng minh.

+, Chứng minh bổ đề 2. Bổ đề 2 được chứng minh bằng phương pháp quynạp bằng cách lý luận tương tự như chứng minh bổ đề 1.Áp dụng hai bổ đề trên với a = b = 2007, ta được

200820072006

+ 200620072008

= S200620072007 + t20082007

2009

=(

S2006 + t200820072)

20072007.

Vì S2006 không chia hết cho 2007 nên số k lớn nhất thỏa mãn điều kiện bài toánlà 2007.

Ví dụ 1.2.12. Dãy số (un) xác định như sau:{

u0 = u1 = 1

un+1 = un−1un + 1; n = 1, 2, . . .

Chứng minh rằng: u2008 − 3...4.

Lời giải: Từ cách xác định dãy số, ta có u2 = 2; u3 = 3; u4 = 7; u5 = 22; u6 =

155; . . . Nhận thấy:

u2 ≡ 2 (mod 4); u5 ≡ 2 (mod 4).

u3 ≡ 3 (mod 4); u4 ≡ 3 (mod 4).

Ta dự đoán:

{

un ≡ 2 (mod 4) nếu n = 3k + 2, (k ∈ Z)

un ≡ 3 (mod 4) nếu n = 3k + 1 hoặc n = 3k, (k ∈ Z).(1.13)

12

Page 17: SÁU PHƯƠNG PHÁP GIẢI CÁC BÀI TOÁN PHỔ THÔNG

Chương 1. Phương pháp quy nạp

Ta sẽ chứng minh quan hệ (1.13) bằng quy nạp.1. Cơ sở quy nạp. Với n = 2, 3, 4, 5, ta có quan hệ (1.13) đúng.2. Quy nạp. Giả sử (1.13) đúng với n = t. Ta cần chứng minh (1.13) đúng với

n = t + 1.+) Nếu t = 3k, khi đó

{

t− 1 = 3k − 1 = 3(k − 1) + 2

t + 1 = 3k + 1.

Do đó, theo giả thiết quy nạp ta có

ut ≡ 3 (mod 4); ut−1 ≡ 2 (mod 4)

Suy raut+1 ≡ 3.2 + 1 ≡ 3 (mod 4),

hay (1.13) đúng.+) Nếu t = 3k + 1, suy ra

{

t− 1 = 3k

t + 1 = 3k + 2 .

Do đó, theo giả thiết quy nạp ta có

ut ≡ 3 (mod 4); ut−1 ≡ 3 (mod 4)

Suy raut+1 ≡ 3.3 + 1 ≡ 2 (mod 4),

hay (1.13) đúng.+) Nếu t = 3k + 2, suy ra

{

t− 1 = 3k + 1

t + 1 = 3(k + 1) .

Do đó, theo giả thiết quy nạp ta có

ut ≡ 2 (mod 4); ut−1 ≡ 3 (mod 4)

Suy raut+1 ≡ 3.2 + 1 ≡ 3 (mod 4),

hay (1.13) đúng. Vậy (1.13) được chứng minh. Do 2008 = 3.669 + 1, nên u2008 ≡ 3

(mod 4), hay u2008 − 3...4. �

13

Page 18: SÁU PHƯƠNG PHÁP GIẢI CÁC BÀI TOÁN PHỔ THÔNG

Chương 1. Phương pháp quy nạp

Ví dụ 1.2.13. Chứng minh rằng số được thành lập bởi 3n chữ số giống nhau thìchia hết cho 3n, trong đó n là một số nguyên dương cho trước.

Lời giải: Gọi A(n) = aa . . . a, trong đó A(n) gồm 3n chữ số a và 1 ≤ a ≤ 9. Tachứng minh khẳng định bằng quy nạp theo n.

1. Cơ sở quy nạp. Với n = 1, ta có A(1) = aaa chia hết cho 31 = 3. Khẳngđịnh đúng với n = 1.

2. Quy nạp. Giả sử A(n) chia hết cho 3n, ta cần chứng minh A(n + 1) chiahết cho 3n+1.

Thật vậy, ta có

A(n+ 1) = A(n)A(n)A(n) = A.102.3n

+ A.103n

+ A

= A(

102.3n

+ 103n

+ 1)

.

Vì 10 ≡ 1 (mod 3) nên 102.3n

+ 103n

+ 1 ≡ 0 (mod 3).Hơn nữa, theo giả thiết quy nạp, A(n) chia hết 3n.Do đó, ta có A(n+ 1) chia hết 3n.3 = 3n+1, hay khẳng định đúng với n+ 1.Theo nguyên lý quy nạp, khẳng định đã cho đúng với mọi n ∈ Z+. �

Ví dụ 1.2.14. Mỗi đầu của đường kính thuộc đường tròn tâm 0 ghi số 1. Sauđó tại trung điểm của mỗi cung nhận được ghi số 2 (tổng của hai số được ghi ởhai đầu của mỗi cung) (Bước 2). Coi bốn điểm ghi số là các điểm chia đườngtròn. Khi đó đường tròn được chia thành bốn cung bằng nhau. Giữa mỗi cungnày lại ghi số 3 (tổng của hai số được ghi ở hai đầu của cung tương ứng) (Bước3). Cứ tiếp tục như vậy. Hỏi sau n bước tổng các số được ghi trên đường tròn làbao nhiêu?

01 1

2

2

3

3

3

3

14

Page 19: SÁU PHƯƠNG PHÁP GIẢI CÁC BÀI TOÁN PHỔ THÔNG

Chương 1. Phương pháp quy nạp

Lời giải: Sau n bước tổng các số trên đường tròn là Sn = 2.3n.Ta sẽ chứng minh công thức trên bằng quy nạp theo n.1. Cơ sở quy nạp. Sau Bước 1, trên đường tròn có bốn (22) số 1, 2, 1, 2. Khi

đóS1 = 1 + 2 + 1 + 2 = 6 = 2.31

2. Quy nạp. Giả sử khẳng định đúng với n = k(k ≥ 1), nghĩa là sau k bướctrên đường tròn đã có 2k+1 số

s1, s2, . . . , s2k+1, . . . , s2k+1, (1.14)

với tổng là Sk = 2.3k.

1 1

2

2

3

3

3

3

Sang bước (k + 1), ta coi 2k+1 điểm đã ghi là các điểm chia, nên đường trònđược thành 2k+1 cung bằng nhau. Do trung điểm của mỗi cung này lại ghi tổngcủa hai số đã ghi ở đầu của mỗi cung, nên mỗi số thuộc dãy (1.14) được xuấthiện đúng hai lần trong các tổng mới (các số được ghi tại bước k + 1). Do đó,tổng các số được ghi trên đường tròn sau k + 1 bước là:

Sk+1 = tổng các số đã ghi sau bước k + tổng các số được ghi tại bước k+1

= Sk + 2Sk = 3Sk = 3.2.3k = 2.3k+1

Khẳng định được chứng minh. �

Ví dụ 1.2.15. Chứng minh rằng trên mặt phẳng n đường thẳng khác nhau cùngđi qua một điểm, chia mặt phẳng thành 2n phần khác nhau.

Lời giải: Bài toán được giải quyết bằng quy nạp.1) Cơ sở quy nạp. Với n = 1, ta có một đường thẳng. Nó chia mặt phẳng

thành hai phần, nên khẳng định đúng.

15

Page 20: SÁU PHƯƠNG PHÁP GIẢI CÁC BÀI TOÁN PHỔ THÔNG

Chương 1. Phương pháp quy nạp

2) Quy nạp. Giả sử với n = k khẳng định đã đúng, nghĩa là k đường thẳngtùy ý cùng đi qua một điểm M đã chia mặt phẳng thành 2k phần khác nhau.

Xét n = k + 1 đường thẳng khác nhau tùy ý cùng đi qua một điểm. Kíhiệu các đường này, một cách tương ứng bằng δ1, δ2, · · · , δk, δk+1. Theo giả thiếtquy nạp k đường thẳng δ1, δ2, · · · , δk đã chia mặt phẳng thành 2k phần khác nhau.

M

δs

δt

δk+1

Vì các đường thẳng đều khác nhau và cùng đi qua điểm M , nên tồn tại cácchỉ số s, t (1 ≤ s, t ≤ k) để δk+1 là đường thẳng duy nhất nằm trong góc đượclập nên bởi δs và δt. Khi đó δk+1 chia hai phần mặt phẳng được giới hạn bởiδs và δt thành bốn phần. Bởi vậy k + 1 đường thẳng δ1, δ2, · · · , δk, δk+1 chia mặtphẳng thành

2k − 2 + 4 = 2k + 2 = 2 (k + 1)

phần khác nhau. Khẳng định được chứng minh. �

16

Page 21: SÁU PHƯƠNG PHÁP GIẢI CÁC BÀI TOÁN PHỔ THÔNG

Chương 2

Phương pháp chứng minh phảnchứng

Chứng minh là một nét đặc trưng của toán học, tạo ra sự khác biệt giữatoán học với các môn khoa học khác. Nắm bắt phương pháp và kĩ thuật chứngminh cũng là yêu cầu bắt buộc đối với học sinh nói chung. Các phương pháp vàkĩ thuật chứng minh rất phong phú: Từ chứng minh trực tiếp đến gián tiếp, từchứng minh bằng quy nạp đến chứng minh bằng phản chứng, từ ví dụ đến phảnví dụ, từ xây dựng đến không xây dựng.

Trong bài luận văn này xin được đề cập đến phép chứng minh phản chứng,một trong những phương pháp chứng minh kinh điển và quan trọng nhất củatoán học. Chứng minh phản chứng có thể nói là một trong những vũ khí quantrọng nhất của toán học. Nó cho phép chúng ta chứng minh sự có thể và khôngthể của một tính chất nào đó. Nó cho phép chúng ta biến thuận thành đảo, biếnđảo thành thuận. Nó cho phép chúng ta lý luận trên những đối tượng mà khôngrõ là có tồn tại hay không.

2.1 Cơ sở lý thuyết

Cơ sở lý thuyết của phương pháp phản chứng là các định luật trong logic:Gọi p, q, r là các mệnh đề toán học nào đó, khi đó

Định lý 2.1.1. (Các định luật cơ bản)1. (Phi mâu thuẫn): p ∧ p = 0

2. (Bài trung):p ∨ p = 1

Định lý 2.1.2. (Các định luật phản chứng)1. (Phản chứng) (p ⇒ q) ⇔ (q ⇒ p).

17

Page 22: SÁU PHƯƠNG PHÁP GIẢI CÁC BÀI TOÁN PHỔ THÔNG

Chương 2. Phương pháp chứng minh phản chứng

2. (Phản chứng suy rộng):

(p ∧ q ⇒ r) ⇔ (p ∧ r ⇒ q) (2.1)

3. (p ⇒ q ∧ q) ⇒ p.

4. (q ⇒ 0) ⇒ q.

Các định luật trên có thể được chứng minh chẳng hạn bằng phương pháplập bảng chân lý hoặc phương pháp biến đổi tương đương. Chẳng hạn, ta có thểchứng minh (2.1) như sau:

(p ∧ q ⇒ r) ⇔ p ∧ q hoặc r (do a ⇒ b ⇔ a hoặc b)

⇔p hoặc q hoặc r (do a ∧ b ⇔ a hoặc b)

⇔(p hoặc r) hoặc q (giao hoán, kết hợp và r ⇔ r)

⇔p ∧ r hoặc q

⇔(p ∧ r ⇒ q) đpcm.

2.2 Nội dung của phương pháp phản chứng

Để chứng minh khẳng định p ⇒ q bằng phương pháp phản chứng ta giả sử q

sai, tức là q là mệnh đề đúng. Nếu từ đó thu được một điều vô lý (vl) thì điềuđó chứng tỏ giả sử của ta là sai, tức là q đúng. Điều vô lý (vl) có thể thuộc mộttrong các dạng sau:

+) Điều trái với giả thiết p (vl ≡ p)

+) Điều trái với một trong các kiến thức đã biết (vl ≡ 0)

+) Điều trái với giả sử phản chứng (vl ≡ q).Khi xây dựng mệnh đề phản chứng q ta cần nhớ1. p ⇔ p.2. p ∧ q ⇔ p hoặc q.3. p ∨ q ⇔ p ∧ q.4. ∀x, p(x) ⇔ ∃x, p(x).5. ∃x, p(x) ⇔ ∀x, p(x).6. ∀x, p(x) ⇔ ∃x, p(x).7. ∃x, p(x) ⇔ ∀x, p(x).8. Nếu p(x) := f(x)Rg(x), thì p(x) là : f(x)Rg(x) trong đó, R và R được xác

định như sau:

18

Page 23: SÁU PHƯƠNG PHÁP GIẢI CÁC BÀI TOÁN PHỔ THÔNG

Chương 2. Phương pháp chứng minh phản chứng

R

R

=

=6=

6= >

>

<

<

Ngoài ra còn có:

Định lý 2.2.1. (Định luật về âm bản). Cho A là một công thức logic mà trongđó chỉ chứa các phép toán: Phủ định, ∧, hoặc đối với các mệnh đề: p1, p2, · · · , pn.Âm bản của A, kí hiệu là Ad là một công thức thu được từ A bằng cách thaypj ∼ pj ; pj ∼ pj,∨ ∼ ∧;∧ ∼ ∨. Khi đó:

A ⇔ Ad

Phương pháp phản chứng còn dựa trên nguyên lý Dirichlet do nhà toán họcĐức nổi tiếng Peter Dirichlet (1805-1859) đề xuất, mà dạng đơn giản nhất củanguyên lý này được phát biểu như sau: "Không thể nhốt 7 chú thỏ vào 3 cáilồng sao cho mỗi lồng không có quá hai chú thỏ". Nói cách khác "Nếu nhốt 7

chú thỏ vào 3 cái lồng, thì phải có ít nhất một lồng có không ít hơn 3 chú thỏ".

2.3 Trình bày lời giải của phương pháp phản chứng

Bài toán: Chứng minh p ⇒ q

Lời giải: Giả sử ngược lại, q sai, tức là q. Mà q ⇒ · · · ⇒ vl. Vậy giả sử của ta làsai, tức là q đúng.

Ngoài ra, ta sẽ giải một số bài toán bằng phương pháp phản chứng dựa trênnguyên lý Dirichlet.

2.4 Một số ví dụ minh họa

Ví dụ 2.4.1. Cho f(x) = ax2 + bx+ c. Giả sử

|a|+ |b|+ |c| > 17 (2.2)

Chứng minh rằng∃x ∈ [0; 1], |f(x)| > 1 (2.3)

Lời giải: Ta sẽ chứng minh bằng phương pháp phản chứng. Giả sử (2.3) sai, tứclà

∀x ∈ [0; 1], |f(x)| ≤ 1 (2.4)

19

Page 24: SÁU PHƯƠNG PHÁP GIẢI CÁC BÀI TOÁN PHỔ THÔNG

Chương 2. Phương pháp chứng minh phản chứng

Chọn x = 0; 12 ; 1, từ (2.4) ta được |c| ≤ 1 và:

{

|a+ b+ c| ≤ 1∣

a4 +

b2 + c

∣ ≤ 1

Suy ra|a|+ |b|+ |c| ≤ 17

Đó là điều vô lý (trái với (2.2)). Vậy giả sử của ta là sai, tức là (2.3) đúng. �

Ví dụ 2.4.2. Chứng minh tập các số nguyên tố P là tập vô hạn.

Lời giải: (phản chứng). Giả sử ngược lại, tập P là hữu hạn. Giả sử P =

{p1, p2, · · · , pn}. Khi đó, tồn tại số nguyên tố lớn nhất. Gọi đó là pn. Tức là:

pn ∈ P và ∀p ∈ P, p ≤ pn.

Xét số x = p1.p2. . . . .pn+1. Ta có, x ∈ N; x không chia hết cho các số p1; p2; . . . ; pn

(vì nếu x...pj nào đó thì 1

...pj: vô lý). Vậy x ∈ P . Mà hiển nhiên, x > pn. Đó là điềuvô lý (trái với cách chọn pn).Vậy điều giả sử của ta là sai, tức là P là tập vô hạn. �

Ví dụ 2.4.3. Có thể chia các số tự nhiên từ 1 đến 21 thành các nhóm đôi một rờinhau, sao cho trong mỗi nhóm số lớn nhất bằng tổng các số còn lại hay không?

Lời giải:Giả sử chia được. Khi đó tổng các số ở mỗi nhóm là một số chẵn (bằng hai

lần số lớn nhất). Vậy tổng của 21 số đã cho là một số chẵn (vì các nhóm đôimột rời nhau và tổng của các số chẵn là số chẵn).

Nhưng tổng của 21 số đó là 21.11 = 231 là số lẻ. Điều vô lý này chứng tỏ giảsử của ta là sai, tức là không chia được thành các nhóm thỏa mãn yêu cầu đềbài.

Ví dụ 2.4.4. Có thể tìm được hay không 5 số nguyên, sao cho các tổng của haisố một trong 5 số đó lập thành 10 số nguyên liên tiếp?

Lời giải:Giả sử tìm được 5 số như vậy. Gọi s là tổng của 5 số đó và n là giá trị nhỏ

nhất của tổng các cặp hai số. Khi đó 10 số nguyên liên tiếp nói trong đề bài là

n, n+ 1, . . . , n+ 9

20

Page 25: SÁU PHƯƠNG PHÁP GIẢI CÁC BÀI TOÁN PHỔ THÔNG

Chương 2. Phương pháp chứng minh phản chứng

Ta tính tổng τ của 10 số đó theo hai cách khác nhau:Một mặt, τ = n+ (n + 1) + (n + 2) + . . .+ (n + 9) = 5 (2n+ 9)

Mặt khác τ = 4s (do trong τ mỗi số đã cho có mặt đúng 4 lần). Từ đó suy ra4s = 5 (2n+ 9) là điều vô lý.Vậy giả sử ban đầu là sai, tức là không thể chọn được 5 số thỏa mãn yêu cầubài ra.

Ví dụ 2.4.5. Cho ba điểm A,B,C phân biệt trên mặt phẳng. Chứng minh rằngnếu tồn tại điểm G thuộc mặt phẳng đó thỏa mãn:

−→GA+

−−→GB +

−→GC =

−→0

thì điểm G đó là duy nhất.

Lời giải: Ta chứng minh bằng phương pháp phản chứng.Giả sử còn có điểm O 6= G thỏa mãn yêu cầu bài toán, tức là

−→OA+

−−→OB +

−→OC =

−→0

Ta có−→GA+

−−→GB +

−→GC =

−→0

⇔ −→GO +

−→OA+

−→GO +

−−→OB +

−→GO +

−→OC =

−→0

⇔ 3−→GO +

−→OA+

−−→OB +

−→OC =

−→0

Do−→OA+

−−→OB +

−→OC =

−→0 , suy ra

3−→GO =

−→0

⇔ −→GO =

−→0

⇔ G ≡ O

Vậy G là duy nhất. �

Ví dụ 2.4.6. (IMO 1982) Cho phương trình

x3 − 3xy2 + y3 = n, n ∈ N∗ (2.5)

1) Chứng minh rằng nếu (2.5) có nghiệm nguyên thì nó không có nghiệmnguyên duy nhất.

2) Tìm nghiệm nguyên của (2.5) khi n = 2005.

21

Page 26: SÁU PHƯƠNG PHÁP GIẢI CÁC BÀI TOÁN PHỔ THÔNG

Chương 2. Phương pháp chứng minh phản chứng

Lời giải:1) Dễ dàng biến đổi

x3 − 3xy2 + y3 = (y − x)3 − 3 (y − x) x2 + (−x)3

= (−y)3 − 3 (−y) (x− y)2 + (x− y)3

Từ đó ta có: Nếu (x; y) là một nghiệm của (2.5) thì (y − x;−x) , (−y; x− y) cũnglà các nghiệm của (2.5). Ba nghiệm này đôi một khác nhau vì nếu có hai nghiệmnào đó bằng nhau thì ta có x = y = 0 là điều vô lý do n > 0.Vậy nếu (2.5) có nghiệm, thì nó có ít nhất ba nghiệm phân biệt, tức là (2.5)

không thể có nghiệm duy nhất.

2) Với n = 2005 giả sử (2.5) có nghiệm (x; y) ∈ Z2. Ta viết các nghiệm theo(mod 3), phương trình đã cho trở thành

x3 + y3 = −1 (mod 3) ⇒ x+ y = −1 (mod 3).

Do vậy ta có các trường hợp sau:

(i)x ≡ 0 (mod 3) và y ≡ −1 (mod 3); (2.6)

(ii)x ≡ 1 (mod 3) và y ≡ 1 (mod 3); (2.7)

(iii)x ≡ −1 (mod 3) và y ≡ 0 (mod 3). (2.8)

Trong trường hợp (i) đặt x = 3m, y = 3n − 1, thay vào phương trình đã cho tađược V T ≡ −1 (mod 3) còn V P = 2005 ≡ −2 (mod 9) ⇒ vô lý.Trong trường hợp (ii), do (y − x;−x) cũng là nghiệm, mà y − x ≡ 0 (mod 3) và−x ≡ −1 (mod 3) nên ta cũng thu được một điều vô lý.Tương tự từ trường hợp (iii) ta cũng thu được một điều vô lý.Vậy với n = 2005 phương trình đã cho không có nghiệm nguyên. �

Ví dụ 2.4.7. (IMO 1983). Tìm tất cả những hàm số f(x) : R+ → R+, là toànánh và thỏa mãn đồng thời hai điều kiện sau:

1) f(xf(y)) = yf(x), ∀x, y ∈ R+.

2) f(x) → 0 khi x → +∞.

Lời giải: Do f(x) là toàn ánh và 1 ∈ R+ nên ∃y0 ∈ R+ : f(y0) = 1. Trong 1) chox = 1, y = y0 ta được

f(1) = f(1.1) = f(1.f(y0)) = y0f(1) → y0 = 1(do f(1) > 0) ⇒ y0 = 1 ⇒ f(1) = 1.

22

Page 27: SÁU PHƯƠNG PHÁP GIẢI CÁC BÀI TOÁN PHỔ THÔNG

Chương 2. Phương pháp chứng minh phản chứng

Vậy f(x) có một điểm bất động là x = 1. Ta sẽ chứng minh đó là điểm bất độngduy nhất.Thật vậy, giả sử f(x) có hai điểm bất động x, y phân biệt, khi đó ta có:

(i) f(xy) = f(xf(y)) = yf(x) = yx = xy → xy cũng là điểm bất động củaf(x).

(ii) 1 = f(1) = f(

1xf(x)

)

= xf( 1x) → 1

xcũng là điểm bất động của f(x).

Giả sử còn có điểm bất động x 6= 1, khi đó theo (ii), f(x) còn có điểm bất động1x

. Trong hai số đó phải có một số lớn hơn 1. Giả sử x > 1. Theo (i), f(x) sẽ cóvô số điểm bất động xn = xn, n ∈ N∗. Ta có dãy đối số (xn) → +∞ (do x > 1), dođó dãy hàm (f(xn)) = (xn) → +∞. Điều đó trái với 2) : f(x) → 0 khi x → +∞.Vậy giả sử của ta là sai, tức là f(x) có duy nhất một điểm bất động là x = 1.Nhưng trong (i) cho y = x ta được f(xf(x)) = xf(x), ∀x ∈ R+ ⇒ xf(x) cũng làđiểm bất động của f(x). Vậy ta phải có

∀x ∈ R+ : xf(x) = 1 ⇔ f(x) =

1

x.

Dễ tìm thấy hàm số này thỏa mãn các điều kiện bài ra.Vậy f(x) = 1

xlà hàm số cần tìm.

Ví dụ 2.4.8. (IMO 1983) Cho các số a, b, c ∈ N∗, đôi một nguyên tố cùng nhau.Chứng minh rằng phương trình

xbc + yca+ zab = 2abc− ab− bc− ca (2.9)

không có nghiệm tự nhiên.

Lời giải: Giả sử (2.9) có nghiệm (x; y; z) ∈ N3. Từ (2.9) ta có (z + 1) ab...c. Mà

(a; c) = (b; c) = 1 ⇒ z + 1...c, mà z + 1 > 0 nên từ đó có z + 1 ≥ c. Tương tự, có

x+ 1 ≥ a, y + 1 ≥ b. Từ các đánh giá này ta được

V T (2.9) ≥ (a− 1) bc + (b− 1) ca + (c− 1) ab

= 3abc− ab− bc− ca

> 2abc− ab− bc− ca = V P (2.9)

Điều này vô lý chứng tỏ giả sử của ta là sai, tức là phương trình (2.9) không cónghiệm tự nhiên. �

Ví dụ 2.4.9. (Putnam 1998) Chứng minh rằng với mọi số nguyên a, b, c luôntìm được số nguyên dương n, sao cho số

f(n) = n3 + an2 + bn + c

23

Page 28: SÁU PHƯƠNG PHÁP GIẢI CÁC BÀI TOÁN PHỔ THÔNG

Chương 2. Phương pháp chứng minh phản chứng

không phải là số chính phương.

Lời giải: Ta chứng minh mệnh đề sau:

∀ (a; b; c) ∈ Z3, ∃n ∈ N

∗ : f(n) không phải là số chính phương. (2.10)

Nhận xét rằng mọi số chính phương đều ≡ 0 (mod 4) hoặc ≡ 1 (mod 4). Giả sử(2.10) là mệnh đề sai, tức là

∃ (a; b; c) ∈ Z3, ∀n ∈ N

∗ : f(n) là số chính phương. (2.11)

Đặc biệt, từ đó ta có

f(1) = a+ b+ c+ 1 là số chính phương(i)f(2) = 4a + 2b+ c+ 8 là số chính phương(ii)f(3) = 9a+ 3b+ c+ 27 là số chính phương(iii)f(4) = 16a+ 4b+ c+ 64 là số chính phương(iv)

Từ (ii) và (iv) ta có f(4) − f(2) = 2b (mod 4). Mà 2b là số chẵn, còn theo nhậnxét thì f(4)− f(2) chỉ có thể ≡ 0, 1,−1 (mod 4) ⇒ 2b ≡ 0 (mod 4).Từ (i) và (iii) ta có f(3)− f(1) ≡ (2b+ 2) (mod 4).Tương tự trên ta cũng có 2b+ 2 ≡ 0 (mod 4) ⇒ 2 ≡ 0 (mod 4) là điều vô lí.Vậy giả sử của ta là sai, tức là (2.10) là mệnh đề đúng. �

Ví dụ 2.4.10. Cho a, b, c là các số thực dương. Chứng minh rằng

a√a2 + 8bc

+b√

b2 + 8ca+

c

c2 + 8ab≥ 1.

Lời giải: Ta sẽ giải bài toán trên bằng phương pháp phản chứng như sau: Đặt

x =a√

a2 + 8bc, y =

b√b2 + 8ca

, z =c

c2 + 8ab

Khi đó, ta có(

1

x2− 1

)

(

1

y2− 1

)

(

1

z2− 1

)

= 64

và ta cần chứng minhx+ y + z ≥ 1

Giả sử ngược lại, x+ y + z < 1. Khi đó

(

1x2 − 1

)

(

1y2 − 1

)

(

1z2

− 1)

>

(

(x+ y + z)2

x2− 1

)(

(x+ y + z)2

y2− 1

)(

(x+ y + z)2

z2− 1

)

=(2x+ y + z) (y + z) (2y + x+ z) (x+ z) (2z + x+ y) (x+ y)

x2y2z2

(2.12)

24

Page 29: SÁU PHƯƠNG PHÁP GIẢI CÁC BÀI TOÁN PHỔ THÔNG

Chương 2. Phương pháp chứng minh phản chứng

Áp dụng bất đẳng thức Cauchy cho các dấu ngoặc trong V P (2.12) ta được:

2x+ y + z = x+ x+ y + z ≥ 4 4√x.x.y.z

2y + x+ z = y + y + x+ z ≥ 4 4√y.y.z.z

2z + x+ y = z + z + x+ y ≥ 4 4√z.z.x.y

y + z ≥ 2√y.z

x+ z ≥ 2√x.z

x+ y ≥ 2√x.y

Thay vào biểu thức (2.12) ta được VP(2.12) > 192, mà theo giả thiết VT(2.12) =

64, do đó ta có (64 > 192) (mâu thuẫn).Vậy điều giả sử của ta là sai. Tức là bài toán được chứng minh. �

Ví dụ 2.4.11. Cho a, b, c là các số thực sao cho a+b+c > 0, ab+bc+ca > 0, abc > 0.Chứng minh rằng a, b, c > 0.

Lời giải: (phản chứng). Giả sử một trong các số a, b, c < 0. Vì ta có abc > 0 nênphải có hai số âm, một số dương. Không mất tính tổng quát, giả sử a, b < 0, c > 0.Khi đó

ab+ bc + ca = a(a + b+ c)− a2 + bc < 0

Mâu thuẫn với điều kiện ban đầu. Do đó giả sử của ta là sai.Vậy ta có điều phải chứng minh. �

Ví dụ 2.4.12. Chứng minh rằng với mỗi số nguyên dương k tồn tại một sốnguyên dương n > 1, sao cho: Ci

n 6 ...k(∀i = 1, n− 1).

Lời giải: (phản chứng). Giả sử Cin

...k. Do đó, lấy k = 4, khi đó tồn tại một sốnguyên dương n > 1 để Ci

n

...4 (∀i = 1, n).Suy ra

n−1∑

i=1

= Cin ≡ 0 (mod 4).

Mặt khácn−1∑

i=1

= Cin = 2n − 2 ≡ −2 (mod 4).

Do đó, ta có mâu thuẫn.Vậy điều giả sử của ta là sai. Ta được điều phải chứng minh. �

25

Page 30: SÁU PHƯƠNG PHÁP GIẢI CÁC BÀI TOÁN PHỔ THÔNG

Chương 2. Phương pháp chứng minh phản chứng

Ví dụ 2.4.13. Xét các số gồm 7 chữ số phân biệt được lập thành từ các số{1, 2, . . . , 7} .

Liệu trong các số được lập có ba số a, b, c mà a+ b = c.

Hỏi có hai số khác nhau a, b mà a...b.

Lời giải: 1. Ta có:

1 + 2 + . . .+ 7 = 28 ≡ 1 (mod 9).

Vậy số bất kỳ trong các số nói trên khi chia cho 9 đều dư 1.Giả sử tồn tại ba số a, b, c mà a + b = c. Khi đó: a + b ≡ 2 (mod 9), còn c ≡ 1

(mod 9).Điều này vô lý, nên không tồn tại ba số thỏa mãn yêu cầu bài toán.

2. Giả sử có hai số a, b mà a...b, tức là ∃n ∈ N∗ : a = nb.

Do b.7 ≥ 1234567.7 > 7654321 ≥ a, nên 7b ≥ nb ⇔ n ≤ 7.Vì a = nb, nên a ≡ n (mod 9)(1 < n ≤ 7).Mặt khác, a ≡ 1 (mod 9). Điều này vô lý, nên trong các số được lập không cóhai số nào, mà một trong hai số chia hết cho số kia.

Ví dụ 2.4.14. Cho hai đường thẳng chéo nhau. Chứng minh rằng có duy nhấtmột mặt phẳng chứa đường thẳng này và song song với đường thẳng kia.

Lời giải: Giả sử ta có hai đường thẳng chéo nhau a và b.lấy điểm M bất kỳ thuộc a. Qua M kẻ đường thẳng b′ song song với b. Gọi (α)là mặt phẳng xác định bởi a và b′.

a

b

M

b′

α

Ta có: b//b′ và b′ ⊂ (α), từ đó suy ra b//(α).Hơn nữa (α) ⊃ a nên (α) là mặt phẳng cần tìm.Ta chứng minh (α) là mặt phẳng duy nhất. Thật vậy, giả sử có một mặt phẳng(β) khác (α), chứa a và song song với b thì khi đó (α), (β) là hai mặt phẳng phânbiệt cùng song song với b nên giao tuyến của chúng là a, phải song song với b.

26

Page 31: SÁU PHƯƠNG PHÁP GIẢI CÁC BÀI TOÁN PHỔ THÔNG

Chương 2. Phương pháp chứng minh phản chứng

Điều này mâu thuẫn với giả thiết a và b chéo nhau.Tương tự, ta có thể chứng minh có duy nhất một mặt phẳng chứa b và songsong với a.Vậy ta được điều phải chứng minh. �

Ví dụ 2.4.15. Chứng minh rằng nếu mặt phẳng (α) chứa hai đường thẳng cắtnhau a, b và a, b cùng song song với mặt phẳng (β) thì (α) song song với (β)

Lời giải: Gọi M là giao điểm của a và b. Vì (α) chứa a mà a song song với (β)nên (α) và (β) là hai mặt phẳng phân biệt. Ta cần chứng minh (α) song songvới (β).Giả sử (α) và (β) không song song và cắt nhau theo giao tuyến c.

M

a

β

c

Ta có

a//(β)

(α) ⊃ a

(α) ∩ (β) = c

⇒ c//a

b//(β)

(α) ⊃ b

(α) ∩ (β) = c

⇒ c//b

.Như vậy từ M kẻ được hai đường thẳng a, b cùng song song với c. Theo định

lý 1, §2 (SGK hình học 11), điều này mâu thuẫn.Vậy (α) và (β) phải song song với nhau.

27

Page 32: SÁU PHƯƠNG PHÁP GIẢI CÁC BÀI TOÁN PHỔ THÔNG

Chương 3

Phương pháp suy luận trực tiếp

3.1 Vài nét về phương pháp suy luận trực tiếp

Các bài toán logic hiểu theo nghĩa tương đối rộng gồm các bài toán logic, cácbài toán không mẫu mực (không có cách giải nhất định) thường có nhiều cáchgiải khác nhau, trong đó có phương pháp suy luận trực tiếp.

Phương pháp suy luận logic đã có từ xa xưa và để giải các bài toán logicngười ta chỉ có duy nhất phương pháp này (sau này mới có thêm các phươngpháp khác). Các bài toán logic đa dạng về đề tài, phong phú về chủng loại đòihỏi chúng ta phải biết suy luận đúng đắn, chặt chẽ trên cơ sở vận dụng kiếnthức cơ bản và kinh nghiệm sống của mình. Vì vậy cần phải luyện tập óc quansát, cách lập luận, cách xem xét khả năng có thể xảy ra của một sự kiện và vậndụng những kiến thức đã học vào các tình huống muôn hình muôn vẻ trong cuộcsống hàng ngày.

Đôi khi để giải những bài toán này, chỉ cần những kiến thức toán học đơngiản, nhưng lại đòi hỏi khả năng chọn lọc trường hợp, suy luận chặt chẽ, rõ ràng.Sự phát triển của toán học, chẳng hạn giải tích tổ hợp, phương pháp quy nạp,phản chứng, góp phần phong phú thêm phương pháp suy luận logic trực tiếp.

Những bài toán trong chương này là những ví dụ không quá phức tạp giúpcho người đọc thấy được những kĩ năng cơ bản của tư duy logic. Thông qua lờigiải giúp người đọc biết cách diễn đạt bằng ngôn ngữ logic những bài toán cùngdạng có thể gặp trong đời sống thực tiễn và bằng cách đó người đọc tìm ra lờigiải dễ dàng nhanh gọn hơn. Mỗi bài toán là một ví dụ điển hình và có thể cónhiều bài toán khác tương tự được diễn đạt dưới nhiều hình thức khác nhau.

Những bài toán giải bằng phương pháp suy luận trực tiếp rất có ích cho ngườihọc toán, dạy toán và những người quan tâm đến logic toán. Nó là bước tiếp

28

Page 33: SÁU PHƯƠNG PHÁP GIẢI CÁC BÀI TOÁN PHỔ THÔNG

Chương 3. Phương pháp suy luận trực tiếp

cận đầu tiên đến logic toán học. Nó rèn luyện tư duy logic, khả năng phản xạ,trí thông minh, là hình thức thể thao trí tuệ phục vụ cho đông đảo người đọcđặc biệt là lứa tuổi học sinh ở nhiều cấp học khác nhau.

Điều cơ bản của phương pháp này là thông qua việc phân tích các điều kiệncủa bài toán, cần tìm ra mối quan hệ logic giữa các mệnh đề.

3.2 Các ví dụ về vận dụng phương pháp suy luậntrực tiếp

Ví dụ 3.2.1. Có ba em chơi trò đội mũ. Em chủ trì giơ ba mũ đỏ, hai mũ xanh,rồi yêu cầu ba em ngồi theo hàng dọc và không nhìn về phía sau, rồi từ phía sauchụp lên đầu mỗi em một cái mũ còn hai cái cất đi. Chứng minh rằng trong mọicách đội đều có một em nhận ra màu mũ của mình.

Lời giải: Giả sử em A ngồi sau em B, em B ngồi sau em C, rõ ràng em A quansát được mũ của em B và C còn em B quan sát được mũ của em C.

- Trước hết do chỉ có hai mũ xanh nên nếu B và C đội mũ xanh thì A đoánđược ngay mũ của mình là mũ đỏ.

- Nếu A im lặng, chứng tỏ trong B và C có một em đội mũ xanh, một emđội mũ đỏ hoặc cả hai em đội mũ đỏ. Khi đó B sẽ phải suy nghĩ và quan sát C.Nếu C đội mũ xanh thì B đoán ngay mình đội mũ đỏ. Nếu B thấy C đội mũ đỏthì B đành im lặng và C sẽ nhận biết ngay mình đội mũ đỏ.

Lời giải của bài toán trên có thể tóm tắt theo sơ đồ sau

A

Đ

im

B

X

C

X

Hình 3.1

Như vậy, trong mọi trường hợp, đều có ít nhất một em đội mũ đỏ. Dấu hiệuđể các em dự đoán được mũ của mình là số mũ xanh mà mình hoặc bạn mìnhquan sát được. Em đoán đúng màu mũ của mình sẽ luôn là em đội mũ đỏ. �

29

Page 34: SÁU PHƯƠNG PHÁP GIẢI CÁC BÀI TOÁN PHỔ THÔNG

Chương 3. Phương pháp suy luận trực tiếp

Ví dụ 3.2.2. (Vị sứ giả thông minh)Một viên quan nước Lỗ đi sang xứ Tề, bị vua Tề tuyên phạt tử hình và bị hànhquyết hoặc chém đầu hoặc treo cổ. Trước khi hành quyết nhà vua cho sứ giả đượcnói một câu, nếu nói đúng thì bị chém đầu, nếu nói sai thì bị treo cổ. Sứ giảmỉm cười nói một câu, nhờ đó đã thoát chết. Bạn hãy cho biết câu nói của sứgiả đó như thế nào?

Lời giải: Vị sứ giả đã nói rằng "Tôi sẽ bị treo cổ". Như vậy, nếu nhà vua đemtreo cổ sứ giả thì sứ giả đó nói đúng. Mà theo điều lệnh sử phạt của nhà vuathì phải đưa sứ giả đi chém đầu (vì ông ta nói đúng). Nếu nhà vua chém đầuthì ông ta đã nói sai. Theo điều lệnh sử phạt của nhà vua thì phải đem sứ giảđi treo cổ.

Thành thử nhà vua không thể hành quyết sứ giả bằng chém đầu cũng nhưtreo cổ, nên sứ giả đã thoát chết.

Ví dụ 3.2.3. Người bản sứ và tên thực dânTrước vành móng ngựa là ba người đàn ông, họ là người bản sứ hoặc tên thựcdân. Quan tòa biết khi được hỏi người bản sứ bao giờ cũng nói thật, còn tênthực dân bao giờ cũng nói dối, nhưng quan tòa không biết ai là người bản sứ, ailà tên thực dân. Quan tòa hỏi người thứ nhất: "Anh là ai?". Nhưng anh ta nóingọng nên quan tòa không hiểu câu trả lời. Hãy xác định câu trả lời của ngườithứ nhất?

Lời giải: Nếu người được hỏi thứ nhất là tên thực dân thì theo bản chất củathực dân, anh ta sẽ trả lời "Tôi là người bản sứ". Nếu người đó là dân bản sứthì theo bản chất của người dân bản sứ anh ta cũng trả lời "Tôi là người bảnsứ".

Như vậy câu trả lời của người thứ nhất chỉ có thể là: "Tôi là người bản sứ".

Ví dụ 3.2.4. a. Trong một căn phòng có 10 người, biết rằng giữa ba người bấtkỳ có hai người quen nhau. Chứng minh rằng, có thể tìm được bốn người mà haingười bất kỳ trong số đó đều quen nhau.b. Khẳng định trên có còn đúng nữa không nếu ở câu a số người trong phòng là9.

Lời giải: a. Giả sử bốn người bất kỳ có hai người không quen nhau. Khi đóA không thể có quá ba người không quen. Nếu A có bốn người không quenthì theo giả thiết giữa bốn người này có hai người không quen và họ cùng với

30

Page 35: SÁU PHƯƠNG PHÁP GIẢI CÁC BÀI TOÁN PHỔ THÔNG

Chương 3. Phương pháp suy luận trực tiếp

A hợp thành bộ ba đôi một không quen nhau. Vậy A có không nhiều hơn bangười không quen, nghĩa là A có không ít hơn sáu người quen. Giả sử A quenvới B1, B2, · · · , B6. Khi đó giữa B1, B2, · · · , B6 không có bộ ba nào đôi một quennhau (Nếu khác thì bộ ba này hợp thành với A thành bộ bốn đôi một quennhau- trái với giả thiết). Hơn nữa có bộ ba mà hai người không quen nhau trongsố sáu người B1, B2, · · · , B6. Chẳng hạn, nếu B1 không quen với B2, B3, B4 thìB2, B3, B4 đôi một quen nhau. Vì thế B1 phải có ít nhất ba người quen trongsố B2, B3, · · · , B6. Khi đó trong số ba người này không tìm được hai người quennhau, ngược lại họ tạo với A và B1 thành bộ bốn người đôi một quen nhau. Tráivới giả thiết, suy ra tồn tại bộ bốn mà hai người bất kỳ quen nhau.b. Ta chứng minh cho khẳng định trên vẫn đúng.

Nếu người nào đó quen với tất cả sáu người thì chứng minh sẽ tương tự phầna. Nếu mỗi người quen có đúng năm người thì tổng số các cặp quen nhau sẽ là9.5/2 không là số nguyên- điều này không thể xảy ra.

Cuối cùng nếu tìm được một người nào đó không quen với ít nhất bốn ngườithì bốn người này phải đôi một quen nhau (nếu khác ta sẽ tìm được bộ ba đôimột không quen nhau) có nghĩa là tạo thành bộ bốn cần tìm- đó là điều phảichứng minh. �

Ví dụ 3.2.5. Mỗi bạn đạt giải mấy trong kỳ thi vô địch toàn quốc?Ba bạn Quân, Hùng, Mạnh vừa đạt giải nhất, nhì, ba trong kì thi học sinh

giỏi toàn quốc. Biết rằng:1. Không có học sinh trường chuyên nào đạt giải cao hơn Quân.2. Nếu Quân đạt giải thấp hơn một bạn nào đó thì Quân không phải là học

sinh trường chuyên.3. Chỉ có duy nhất một bạn không học trường chuyên.4. Nếu Hùng hoặc Mạnh đạt giải nhì, thì Mạnh đạt giải cao hơn bạn quê ở

Hải Phòng.Hãy cho biết: Mỗi bạn đã đạt được giải nào? Bạn nào không học trường

chuyên và bạn nào quê ở Hải Phòng?

Lời giải: Ta nhận xét rằng: Nếu Quân đạt giải nhì hoặc ba, thì theo (2) Quânkhông học trường chuyên. Ta suy ra, theo (3) Hùng và Mạnh học trường chuyên,thành thử theo (1) Quân đạt giải nhất. Điều này vô lý. Vậy Quân phải đạt giảinhất. Trong hai bạn Hùng và Mạnh một người đạt giải nhì và một người đạtgiải ba. Mạnh không thể đạt giải ba (vì theo (4) Mạnh còn đạt giải cao hơn bạnquê ở Hải Phòng).

31

Page 36: SÁU PHƯƠNG PHÁP GIẢI CÁC BÀI TOÁN PHỔ THÔNG

Chương 3. Phương pháp suy luận trực tiếp

Vậy Mạnh phải đạt giải nhì, Hùng đạt giải ba. Đồng thời ta cũng suy raQuân không học trường chuyên và Hùng quê ở Hải Phòng.

Ví dụ 3.2.6. Ai đã nói đùa?Nhà trường cử thầy Nghiêm dẫn bốn học sinh Lê, Huy, Hoàng, Tiến đi thi

đấu điền kinh. Kết quả ba em đạt giải nhất, nhì, ba và một em không đạt giải.Khi về trường mọi người hỏi kết quả các em trả lời như sau:

Lê: Mình đạt giải nhì hoặc ba.Huy: Mình đã đạt giải.Hoàng: Mình được giải nhất.Tiến: Mình không được giải.Nghe xong thầy Nghiêm mỉm cười và nói: "Chỉ có ba bạn nói thật còn một

bạn đã nói đùa "Hãy cho biết học sinh nào đã nói đùa và ai đạt giải nhất, ai không đạt giải?

Lời giải:1) Nếu Lê nói đùa thì ba bạn Huy, Hoàng, Tiến nói thật. Như vậy Lê và

Hoàng cùng đạt giải nhất. Điều này vô lý. Vậy Lê phải nói thật.2) Nếu Huy nói đùa thì Huy không được giải và cả ba bạn còn lại đều nói

thật. Như vậy cả Huy, Tiến đều không đạt giải. Điều này trái với giả thiết củađầu bài. Vậy Huy phải nói thật.

3) Nếu Tiến nói đùa thì Tiến đạt giải và cả ba bạn còn lại đều đạt giải. Nhưvậy cả bốn bạn đều đạt giải. Điều này trái với giả thiết. Như vậy Tiến nói thật.

Vậy Hoàng đã nói đùa. Có nghĩa là Hoàng đã đạt giải nhì hoặc ba cho nênHuy đạt giải nhất. Còn Tiến không đạt giải.Kết luận: Huy đạt giải nhất, Tiến không đạt giải và Hoàng nói đùa.

Ví dụ 3.2.7. Điều mâu thuẫn ở đâu?Trong một tòa nhà chỉ có những cặp vợ chồng và những con nhỏ chưa lập gia

đình. Ban điều tra dân số yêu cầu báo cáo về số người sống trong tòa nhà, đạidiện là một anh thợ thích đùa báo cáo như sau:

Sống trong tòa nhà bố mẹ nhiều hơn con cái. Mỗi con trai đều có một chị hayem gái. Số con trai nhiều hơn số con gái. Mỗi cặp vợ chồng đều có con.

Người ta không thể chấp nhận được báo cáo đó (dù là đùa vui) vì trong đó cómâu thuẫn. Hãy chỉ ra điều mâu thuẫn trong báo cáo trên?

Lời giải: Vì mỗi gia đình đều có con, mỗi con trai đều có một chị gái hay emgái, nên tất cả các gia đình đều có con gái. Suy ra số con gái ít nhất bằng số giađình.

32

Page 37: SÁU PHƯƠNG PHÁP GIẢI CÁC BÀI TOÁN PHỔ THÔNG

Chương 3. Phương pháp suy luận trực tiếp

Mặt khác, số con trai nhiều hơn số con gái, nên tổng số con nhiều hơn hailần số gia đình, hay nhiều hơn số bố mẹ, điều này cho ta thấy mâu thuẫn trongbáo cáo của anh thợ thích đùa ở câu đầu tiên "bố mẹ nhiều hơn con cái" vớicác câu tiếp theo.

Ví dụ 3.2.8. Khi tổ chức múa hát tập thể một giáo viên đã xếp 20 nữ sinh vàmột số nam sinh thành vòng tròn sao cho đối diện với một nữ sinh qua tâmvòng tròn là một nam sinh. Hỏi trên vòng tròn này có hai nam sinh nào đứngkề nhau hay không?

Lời giải: Giả sử không có hai nam sinh nào đứng kề nhau, vì vậy cũng khôngcó hai nữ sinh nào đứng kề nhau. Do đó các bạn nam sinh và nữ sinh được xếpxen kẽ nhau trên vòng tròn, suy ra có tất cả là 20 nam sinh.

Lấy hai bạn nam và nữ đứng ở vị trí đối xứng nhau qua tâm vòng tròn. Bắtđầu từ nữ sinh này ta đánh số các bạn đứng trên một nửa vòng tròn cho đếnnam sinh đối diện. Rõ ràng bạn nữ này được đánh số 1 và bạn nam đối diệnđánh số 21. Khi đó các bạn nữ lần lượt được đánh số 1, 3, 5, · · · , 19, còn các bạnnam được đánh số 2, 4, 6, · · · , 20. Do đó có hai bạn nam sinh được đánh số 20 và21 đứng kề nhau. Điều này mâu thuẫn với giả thiết ban đầu.

Vậy phải luôn luôn có hai nam sinh đứng kề nhau.

Ví dụ 3.2.9. Trong tám viên bi có bề ngoài hoàn toàn giống nhau có một viên binặng hơn. Bằng hai lần cân trên đĩa (không được dùng quả cân). Hãy xác địnhviên bi nặng đó?

Lời giải: Đầu tiên chia tám viên bi thành ba đống theo số lượng 3, 3, 2. Tronglần cân thứ nhất ta bỏ mỗi bên đĩa cân một đống gồm ba viên bi. Có hai khảnăng xảy ra:

a) Nếu cân thăng bằng. Thì viên bi nặng nằm trong đống hai viên bi.Khi đó, ta tiến hành cân hai viên bi ở đống gồm hai viên. Mỗi bên đĩa bỏ

một viên bi ở đống hai viên bi. Viên bên chìm chính là viên bi nặng hơn.b) Nếu cân không thăng bằng, thì bên chìm chứa viên bi nặng hơn.Khi đó ta tiến hành cân lần thứ hai. Bỏ mỗi đĩa một viên bi trong ba viên

bên chìm.Nếu cân thăng bằng, thì viên bi nặng hơn chính là viên bi còn lại nằm trên

đĩa chìm.Nếu cân không thăng bằng, thì viên bi nặng hơn chính là viên bi bên chìm.Vậy chỉ bằng hai lần cân ta đã xác định được viên bi nặng hơn.

33

Page 38: SÁU PHƯƠNG PHÁP GIẢI CÁC BÀI TOÁN PHỔ THÔNG

Chương 3. Phương pháp suy luận trực tiếp

Ví dụ 3.2.10. Học sinh lớp 11 và 12 tổ chức thi đấu cờ với nhau. Số học sinhlớp 11 tham gia gấp 10 lần số học sinh lớp 12 tham gia thi đấu xong điểm họcsinh lớp 11 gấp 4, 5 lần số điểm học sinh lớp 12.

Hãy tính xem có bao nhiêu học sinh tham gia đấu cờ. (Nội quy thi đấu là mỗingười thi đấu một lần với tất cả những người còn lại, người thắng ghi 1 điểmngười thua ghi 0 điểm). Biết rằng tất cả các trận đấu không có trận nào hòa.

Lời giải: Gọi số học sinh lớp 12 tham gia đấu cờ là x. Khi đó số học sinh lớp11 tham gia đấu cờ là 10x và có tất cả là 11x em tham gia đấu cờ.

Số trận đấu có tất cả là 11x(11x−1)2 trận và cũng có chừng ấy điểm thắng.

Số điểm thắng của học sinh lớp 11 đạt được là

4, 5

5, 5× 11x (11x− 1)

2= 4, 5x (11x− 1)

Các em học sinh lớp 11 sẽ thi đấu với nhau 10x(10−1)2 trận và sẽ có số điểm là

5x (10x− 1)

Hiển nhiên ta phải có

4, 5x(11x− 1) ≥ 5x(10x− 1)

99x2 − 9x ≥ 100x2 − 10x

x ≥ x2.

Điều này chỉ xảy ra khi x = 1. Vậy có 11 em tham gia đấu cờ.

34

Page 39: SÁU PHƯƠNG PHÁP GIẢI CÁC BÀI TOÁN PHỔ THÔNG

Chương 4

Phương pháp đồ thị

Rất nhiều bài toán có thể giải bằng cách đưa về bài toán trên đồ thị rồi suyra đáp án.

4.1 Một số khái niệm và kết quả cơ bản của líthuyết đồ thị

Trên mặt phẳng hay trong không gian lấy n điểm. Giữa một số cặp điểmđược nối bằng những đoạn thẳng hay đoạn cong được định hướng hoặc không.Người ta gọi hình nhận được là dạng biểu diễn hình học của đồ thị hay một đồthị. Các điểm đã chọn được gọi là đỉnh của đồ thị. Các đoạn thẳng hay đoạncong đã nối được gọi là cạnh của đồ thị.

Nếu cạnh a nối giữa hai điểm A,B thì A,B được gọi là các đỉnh của cạnh a.Cặp đỉnh x, y được gọi là hai đỉnh kề nhau, nếu chúng khác nhau và là hai

đầu của cùng một cạnh.Dãy α các đỉnh

x1, x2, · · · , xi, xi+1, · · · , xm−1, xm

được gọi là một đường, nếu với mọi chỉ số i(1 ≤ i ≤ m− 1) đều có xi và xi+1 làhai đỉnh kề nhau. Các đỉnh x1, xm được gọi là các đỉnh đầu của đường α. Ngườita còn nói rằng đường α nối giữa đỉnh x1 và đỉnh xm.

Chu trình là một đường có hai đầu trùng nhau.Chu trình mà nó đi qua mỗi đỉnh không quá một lần được gọi là chu trình

sơ cấp.Chu trình (α) được gọi là chu trình Hamilton, nếu nó đi qua tất cả các đỉnh

của đồ thị và qua mỗi đỉnh đúng một lần.

35

Page 40: SÁU PHƯƠNG PHÁP GIẢI CÁC BÀI TOÁN PHỔ THÔNG

Chương 4. Phương pháp đồ thị

Đồ thị G được gọi là đồ thị liên thông, nếu mỗi cặp đỉnh của nó đều có đườngnối với nhau.

Đồ thị G được gọi là đồ thị đầy đủ nếu mỗi cặp đỉnh của nó được nối vớinhau bằng đúng một cạnh.

Số cạnh xuất phát từ đỉnh x được gọi là bậc của đỉnh x.Cây là đồ thị liên thông và không có chu trình.Trong cây T tách ra một đỉnh được gọi là đỉnh gốc, còn các đỉnh có bậc bằng

1 và không phải gốc được gọi là lá hay đỉnh ngọn.

Định lý 4.1.1. Đồ thị mà trong đó tổng bậc của hai đỉnh tùy ý đều không nhỏhơn số đỉnh của đồ thị, liên thông.

Định lý 4.1.2. Đồ thị mà trong đó bậc của mỗi đỉnh đều không nhỏ hơn 2, luônluôn có chu trình sơ cấp.

Hệ quả 4.1.1. Nếu trong đồ thị có đúng 2 đỉnh bậc 1, các đỉnh khác có bậckhông nhỏ hơn 2, thì trong G có đường nối giữa hai đỉnh bậc 1.

Định lý 4.1.3. Đồ thị, mà trong đó tổng bậc của hai đỉnh tùy ý không nhỏ hơnsố đỉnh của đồ thị, luôn luôn có chu trình Hamilton.

Định lý 4.1.4. Trong một đồ thị tùy ý số đỉnh, mà mỗi đỉnh có bậc lẻ, luônluôn là một số chẵn.

Định lý 4.1.5. Cho dãy số nguyên dương a1 = 2, a2 = 5, · · · , an+1 = (n+1)an+1.Khi đó đồ thị đầy đủ an+1 đỉnh với các cạnh được tô bằng n màu luôn luôn cótam giác cùng màu (chu trình gồm ba cạnh cùng màu).

Định lý 4.1.6. Cho dãy số nguyên b2 = 3, b3 = 6, bn+1 = (bn−1)n+2. Đồ thị đầyđủ G với bn+1 − 1 đỉnh (n ≥ 2) và các cạnh được tô bằng n màu, sao cho khôngcó tam giác cùng màu, thì trong đồ thị G có hình 5 cạnh với các cạnh cùng màuvà các đường chéo được tô các màu khác.

4.2 Phương pháp đồ thị

Để giải bài toán T bằng cách thông qua đồ thị, cần thực hiện lần lượt haibước sau

36

Page 41: SÁU PHƯƠNG PHÁP GIẢI CÁC BÀI TOÁN PHỔ THÔNG

Chương 4. Phương pháp đồ thị

4.2.1 Xây dựng đồ thị mô tả các quan hệ

Lấy các điểm trên mặt phẳng hoặc trong không gian tương ứng với các đốitượng đã cho trong bài toán. Dùng ngay các kí hiệu đối tượng để ghi trên điểmtương ứng...

Cặp điểm x, y được nối với nhau bằng một cạnh với "đặc điểm t", khi và chỉkhi các đối tượng x, y có quan hệ (t) với nhau. Khi đó bài toán T đã được chuyểnvề bài toán D trên đồ thị.

4.2.2 Dựa vào các kết quả của lý thuyết đồ thị hoặc lý luậntrực tiếp suy ra đáp án của bài toán D

Nếu đáp án của bài toán D còn dưới dạng "ngôn ngữ đồ thị", thì căn cứ vàophép đặt tương ứng khi xây dựng đồ thị mà diễn đạt thành đáp án bằng ngônngữ thông thường (tức là đáp án của bài toán T ).

4.3 Một số ví dụ

Ví dụ 4.3.1. Trong một cuộc thi đấu bóng bàn An và Bình quy ước với nhau:Người thắng cuộc là người đầu tiên thắng ba ván hoặc thắng hai ván liên tiếp.Hãy xác định số khả năng có thể xảy ra?

Lời giải: Dùng A để kí hiệu An thắng, B để kí hiệu Bình thắng. Dùng cây đểmô tả toàn bộ hiện trạng có khả năng xảy ra.

Xây dựng cây: Xuất phát từ điểm S.Ván đầu tiên có hai khả năng xảy ra: An thắng hoặc Bình thắng, nên lấy hai

điểm sao cho hai điểm này với S không thẳng hàng. Một trong hai điểm này ghiA, còn điểm kia ghi B. Nối S với A bằng một đoạn thẳng hoặc một đoạn congbiểu thị A thắng. Tương tự, để biểu thị B thắng nối S với B bằng một đoạnthẳng hoặc một đoạn cong.

Ván thứ hai lại có hai khả năng: An thắng hoặc Bình thắng, nên xuất pháttừ A cũng lấy hai điểm mới và ghi các kí hiệu tương ứng A,B và từ A kẻ haiđoạn thẳng hoặc hai đoạn cong tới hai điểm mới thêm. Đối với điểm B cũngchọn thêm hai đỉnh mới ghi A và B, rồi từ B kẻ hai đoạn thẳng hay hai đoạncong tới hai điểm mới thêm.

Tiếp theo thực hiện kéo dài các đường một cách tương tự, nhưng do quy ướccủa An và Bình những đường mà trên đó xuất hiện hoặc hai đỉnh liên tiếp ghi

37

Page 42: SÁU PHƯƠNG PHÁP GIẢI CÁC BÀI TOÁN PHỔ THÔNG

Chương 4. Phương pháp đồ thị

cùng bằng một kí hiệu hoặc có ba đỉnh được ghi bằng cùng một kí hiệu đềukhông được kéo dài.

S

A

B

B

B

B

B

B

BB

B

A

A

A

A

A

AA

A

Hình 4.1

Vì An và Bình đấu với nhau năm ván, thì hoặc có người thắng hai ván liêntiếp hoặc có người thắng ba ván. Do đó những đường xuất phát từ S đều khôngcó quá năm cạnh. (Hình 4.1)

Cây có 10 đỉnh ngọn nên có 10 khả năng xảy ra.

Ví dụ 4.3.2. Trên đảo có một số cụm dân cư. Mỗi cụm dân cư có hai đườnglớn và ba đường mòn đi ra. Mỗi đường lớn cũng như đường mòn đều dẫn tớimột cụm dân cư khác. Hai cụm dân cư khác nhau bất kì được nối liền bằng hoặcđường lớn hoặc đường mòn. Hỏi trên đảo này có bao nhiêu đường mòn, bao nhiêuđường lớn?

Lời giải: Trước hết ta cần khẳng định rằng trên đảo có sáu cụm dân cư.Thật vậy, mỗi cụm dân cư ta biểu diễn bằng một điểm. Hai cụm dân cư có

đường lớn (đường mòn) nối với nhau, thì hai cụm tương ứng được nối bằng mộtđường nét liền (đường nét đứt). Vì xuất phát từ mỗi cụm dân cư có hai đường

38

Page 43: SÁU PHƯƠNG PHÁP GIẢI CÁC BÀI TOÁN PHỔ THÔNG

Chương 4. Phương pháp đồ thị

lớn và ba đường mòn đi ra, nên xuất phát từ mỗi điểm đã chọn, chẳng hạn A,có hai đường nét liền và ba đường nét đứt. Bởi vậy mỗi điểm đã chọn A đượcnối với các điểm khác B,C,D,E, F bằng một đường nét liền hoặc một đường nétđứt. Như vậy phải có ít nhất sáu cụm dân cư (6 điểm đã chọn). Mặt khác haicụm dân cư tùy ý đều phải có đường nối với nhau, nên mỗi điểm đều chỉ có thểnối tới với năm điểm. Do đó không còn điểm nào ngoài sáu điểm A,B,C,D,E, F .Bởi vậy, có sáu đường lớn và chín đường nhỏ.

Căn cứ vào yêu cầu của bài toán ta có thể đưa ra một vài khả năng xảy racác đường nối giữa các cụm dân cư như hình 4.2.

B

C

D

E

F

A A

B

C C

D

E

F

Hình 4.2

Ví dụ 4.3.3. Tại một giải bóng đá có bốn đội Anh, Đan Mạch, Hà Lan, ThụyĐiển vào bán kết. Có mấy dự đoán xếp hạng như sau

1. Đan Mạch vô địch, Thụy Điển nhì.2. Đan Mạch nhì, Hà Lan ba.3. Anh nhì, Hà Lan tư.Kết quả là mỗi dự đoán đúng được một đội. Hãy cho biết kết quả xếp hạng

của các đội?

Lời giải: Dùng xi để kí hiệu đội x được xếp hạng i(1 ≤ i ≤ 4). Ta vẽ cây, hainhánh đầu tiên ứng với dự đoán thứ nhất là D1, T2. Từ mỗi nhánh này ta lại cóhai nhánh tương ứng với dự đoán thứ hai. Tiếp tục rẽ nhánh với dự đoán thứba.

Ta chọn đường đi từ gốc 0 tới các điểm thỏa mãn các điều kiện:

• Một đội không thể được xếp hai hạng khác nhau.

• Hai đội không thể xếp cùng một hạng.

39

Page 44: SÁU PHƯƠNG PHÁP GIẢI CÁC BÀI TOÁN PHỔ THÔNG

Chương 4. Phương pháp đồ thị

Suy ra chỉ có đường đi D1H3A2 thỏa mãn.

O

D1

D2

A2 A2 A2A2

H4 H4 H4 H4

H3H3D2

T2

Hình 4.3

Đường tô đậm D1H3A2 thỏa mãn điều kiện mỗi dự đoán đúng được một độimà thứ tự ghi trên đường. Vậy kết quả xếp hạng như sau:

Đan Mạch vô địch.Anh nhì.Hà Lan ba.Còn lại là Thụy Điển thứ tư.

Ví dụ 4.3.4. Cho 6 số nguyên dương tùy ý. Chứng minh rằng luôn có thể chọnra được 2 bộ 3 số mà trong mỗi bộ, từng đôi một đều là nguyên tố cùng nhauhoặc đều không nguyên tố cùng nhau.

Lời giải:Bước 1: Chuyển bài toán sang bài toán về đồ thị màu G(X,E).

- Đỉnh: Cho tương ứng mỗi số với một đỉnh X = {A,B,C,D,E, F}- Cạnh: Đoạn nối giữa hai đỉnh tương ứng với hai số

• Nguyên tố cùng nhau được tô màu xanh (đường nét đứt).

• Không nguyên tố cùng nhau được tô màu đỏ (đường nét liền).

Khi đó, ta được bài toán đồ thị màu: Trong đồ thị đầy đủ G(X, E) có 6 đỉnhvới hai cạnh màu, thì luôn luôn có thể tìm được hai tam giác với cạnh cùngmàu.

Giả sử đỉnh A là mút của AB,AC,AD cùng màu đỏ (đường nét liền) nhưhình vẽ dưới đây

40

Page 45: SÁU PHƯƠNG PHÁP GIẢI CÁC BÀI TOÁN PHỔ THÔNG

Chương 4. Phương pháp đồ thị

A

B

C

DHình 4.4

Ta xét tam giác được lập nên từ ba đỉnh đối của A là BCD. Trong tam giácBCD hai khả năng có thể xảy ra:

1) Có ít nhất một cạnh màu đỏ. Chẳng hạn cạnh BC màu đỏ, khi đó tamgiác ABC có cạnh màu đỏ. (Hình 4.5a)

2) Tam giác BCD không có cạnh nào màu đỏ. Nghĩa là tam giác BCD cócạnh đều là màu xanh. (Hình 4.5b)

A

BC

F

E

D

A

B

D

C

Hình 4.5a Hình 4.5b

Vậy với mọi trường hợp, trong đồ thị G đều có tam giác cùng màu (theoĐịnh lý (4.1.5))

Giả sử rằng trong G có tam giác màu đỏ, chẳng hạn tam giác ABC màu đỏ.Ta chứng minh tiếp G còn có tam giác thứ hai nữa với các cạnh cùng màu.

Nếu trong G, ta tạm thời không xét đến một đỉnh của tam giác ABC, chẳnghạn đỉnh A cùng tất cả các cạnh thuộc nó. Ta được đồ thị con đầy đủ G1 có 5

đỉnh.Nếu trong G1 có tam giác cùng màu, thì bài toán đã được giải xong.

41

Page 46: SÁU PHƯƠNG PHÁP GIẢI CÁC BÀI TOÁN PHỔ THÔNG

Chương 4. Phương pháp đồ thị

Ngược lại, trong G1 không có tam giác cùng màu, thì theo Định lý (4.1.6) vớin = 2, ta có thể biểu diễn G1 thành một hình 5 cạnh với các cạnh màu đỏ vàđường chéo màu xanh.

Bây giờ, ta khôi phục lại đỉnh thứ 6 A và các cạnh màu thuộc nó. Xét haicạnh AD và AF nếu chúng đều màu xanh thì ta có tam giác mới ADF có màuxanh. Nếu AD hoặc AF màu đỏ thì ta có tam giác màu đỏ nữa được hình thànhlà ABD hoặc ACF . Vậy ngoài tam giác ABC, ta có thêm một tam giác nữa cócác cạnh cùng màu. Khẳng định đã được chứng minh.

C

F

E

D

BC B

D

E

F

AHình 4.6

Bước 3: Thuyết minh lời giảiTrong G luôn tồn tại hai tam giác với cạnh cùng màu. Nếu cả hai tam giác

đều màu đỏ, thì ta có hai bộ ba số, mà trong mỗi bộ chúng đôi một nguyên tốcùng nhau. Nếu chỉ có một tam giác màu đỏ, thì ta được một bộ ba số đôi mộtnguyên tố cùng nhau, và một bộ ba số đôi một không nguyên tố cùng nhau.Nếu cả hai tam giác màu xanh, nghĩa là ta được hai bộ ba số, mà trong mỗi bộ,chúng đôi một không nguyên tố cùng nhau.

Ví dụ 4.3.5. Để mừng con đạt giải trong kì thi toán quốc tế lần thứ 42, một giađình dự định mời tiệc. Trong số khách mời:a) Người chồng muốn có ít nhất ba người từng đôi một quen nhau.

42

Page 47: SÁU PHƯƠNG PHÁP GIẢI CÁC BÀI TOÁN PHỔ THÔNG

Chương 4. Phương pháp đồ thị

b) Người vợ lại muốn có ít nhất bốn người từng đôi một chưa quen nhau.Hỏi họ phải mời ít nhất bao nhiêu bạn, để mong muốn của chồng của vợ đượcthỏa mãn?

Lời giải:Bước 1: Xây dựng đồ thị G(X,E) mô tả quan hệ của các đối tượng.

• Đỉnh: Mỗi khách cho tương ứng với một đỉnh.

• Cạnh: Đoạn nối giữa hai đỉnh tương ứng với hai khách:

– Quen nhau được tô màu xanh (nét liền).

– Không quen nhau tô màu đỏ (nét đứt).

Khi đó đồ thị G(X,E) mô tả quan hệ quen biết của các khách dự kiến mời.Theo cách xây dựng trên, để thực hiện yêu cầu của người chồng, thì trong sốkhách mời phải có 3 người quen nhau từng đôi một, nên đồ thị G(X,E) phảicó ít nhất một tam giác màu xanh, còn để thỏa mãn yêu cầu của người vợ, thìtrong số khách mời phải có ít nhất 4 người từng đôi một không quen biết nhau,nên trong G(X,E) phải có ít nhất một tứ giác mà các cạnh và các đường chéocủa nó phải có màu đỏ. Do đó bài toán được dẫn về bài toán trên đồ thị: Tìmsố đỉnh ít nhất để đồ thị đầy đủ G(X,E) với các cạnh màu xanh hoặc đỏ mà

a) Hoặc có ba đỉnh được nối với nhau bởi cạnh màu xanh,b) Hoặc bốn đỉnh từng đôi một được nối với nhau bởi cạnh màu đỏ.Bước 2: Giải bài toán đồ thị G1) Nếu G có 8 đỉnh thì có thể xảy ra trường hợp mà cả (a) lẫn (b) không thỏa

mãn. Như cách tô màu G ở hình 4.7, thì không có tam giác nào xanh, hoặc tứgiác nào với cạnh và đường chéo cùng màu đỏ.

43

Page 48: SÁU PHƯƠNG PHÁP GIẢI CÁC BÀI TOÁN PHỔ THÔNG

Chương 4. Phương pháp đồ thị

x1

x2

x3

x4

x5

x6

x7

x8

Hình 4.7

2) Xét G(X,E) đầy đủ với 9 đỉnh, các cạnh được tô bằng hai màu xanh, đỏ.

X = {x1, x2, . . . , x8, x9}

Hai khả năng có thể xảy ra:10) Có 4 cạnh màu xanh xuất phát từ một đỉnh.Chẳng hạn x1 có 4 cạnh x1x2, x1x3, x1x4, x1x5 màu xanh. Khi đó nếu trong các

cạnh nối đôi một giữa 4 đỉnh x2, x3, x4, x5 có một cạnh xanh, giả sử cạnh x3x4

xanh, thì có tam giác x1x3x4 cạnh xanh hình 4.8 a được thỏa mãn, nếu khôngthì tứ giác x2x3x4x5 có cạnh và các đường chéo màu đỏ hình 4.8 b, (b) được thỏamãn.

44

Page 49: SÁU PHƯƠNG PHÁP GIẢI CÁC BÀI TOÁN PHỔ THÔNG

Chương 4. Phương pháp đồ thị

x1

x2

x3

x4

x5

x1

x2

x3

x4

x5Hình 4.8 a Hình 4.8 b

20) Có nhiều nhất là 3 cạnh màu xanh xuất phát từ mỗi đỉnh.Bởi vậy, phải có ít nhất một đỉnh, x1 chẳng hạn, là đầu mút của không quá 2

cạnh màu xanh (vì nếu mỗi đỉnh là đầu mút của đúng 3 cạnh màu xanh, thì sốcạnh màu xanh là 3×9

2 /∈ N). Khi đó, tại x1 phải là đầu mút của ít nhất 6 cạnhđỏ. Giả sử 6 cạnh đỏ đó là x1x4, x1x5, x1x6, x1x7, x1x8, x1x9.

Xét đồ thị con đầy đủ G1 có 6 đỉnh x4, x5, . . . , x9 với 2 màu cạnh. Theo địnhlý (4.1.5) với n = 2, trong G1 có tam giác cùng màu. Nếu là tam giác màu xanh,thì (a) được thỏa mãn, nếu là tam giác màu đỏ thì 3 đỉnh này cùng với đỉnh x1

tạo thành đồ thị con đầy đủ có 4 đỉnh với các cạnh đều màu đỏ, (b) được thỏamãn.

x1

x2x3

x4

x5

x6

x7

x8x9Hình 4.9

Bước 3: Chuyển về ngôn ngữ xuất phát.

45

Page 50: SÁU PHƯƠNG PHÁP GIẢI CÁC BÀI TOÁN PHỔ THÔNG

Chương 4. Phương pháp đồ thị

Đồ thị đầy đủ với 9 đỉnh, các cạnh được tô bằng hai màu xanh, đỏ thì luôncó hoặc tam giác xanh hoặc tứ giác mà các cạnh và đường chéo đều màu đỏ,nhưng với 8 đỉnh thì tính chất trên không tồn tại. Vậy số đỉnh của đồ thị G phảiít nhất bằng 9, nên số khách ít nhất họ phải mời là 9, thì mong muốn của vợhoặc chồng được thỏa mãn.

Ví dụ 4.3.6. Lấy n(n ≥ 4) số nguyên dương khác nhau tùy ý, sao cho cứ 4 sốbất kỳ có ít nhất một số có ước chung với 3 số còn lại. Chứng minh rằng trongcác số đã chọn ra có ít nhất n− 3 số, mà mỗi số này có ước chung với tất cả cácsố đã chọn ra.

Lời giải:Bước 1: Xây dựng đồ thị mô tả các quan hệ.Đỉnh: Lấy n điểm trên mặt phẳng hay trong không gian tương ứng với các

số đã chọn ra và ghi các số này lên các điểm tương ứng.Cạnh: Cặp điểm x, y được nối bằng một đoạn thẳng hay đoạn cong không đi

qua các điểm tương ứng trung gian khác khi và chỉ khi cặp x, y có ước chung.Đồ thị G nhận được mô tả toàn bộ quan hệ có ước chung giữa các số đã chọnra.

Bước 2: Chứng minh trong đồ thị G có ít nhất n− 3 đỉnh có bậc n− 1

a) Nếu trong G có hai cặp đỉnh không kề nhau, thì chúng phải có đỉnh chung.Thật vậy, giả sử trong G có hai cặp đỉnh không kề nhau, chẳng hạn A,B và C,D

nhưng chúng không có đỉnh chung. Khi đó trong 4 đỉnh này không có một đỉnhnào kề với 3 đỉnh còn lại. Ta đi tới mâu thuẫn với giả thiết. Vậy hai cặp A,B vàC,D phải có một đỉnh chung chẳng hạn B ≡ D.

b) Trong đồ thị G có không ít hơn n− 3 đỉnh có bậc n− 1

- Nếu trong G tất cả các đỉnh đều kề nhau từng đôi một, thì cả n đỉnh củaG đều có bậc n− 1.

- Nếu trong G có duy nhất một cặp đỉnh không kề nhau, thì G có n− 2 đỉnhbậc n− 1.

- Giả sử G có hai cặp đỉnh không kề nhau, khi đó theo phần a) hai cặp đỉnhnày phải có đỉnh chung. Giả sử các đỉnh đó là A,B,C. Nếu sử dụng đường nétđứt để biểu thị tính không kề nhau của các đỉnh ta có hình 4.10a.

Kết nạp đỉnh E tùy ý vào nhóm A,B,C. Khi đó trong bộ 4: A,B,C,E phảicó ít nhất một đỉnh kề với 3 đỉnh còn lại. Đỉnh này không thể là các đỉnh A,B,C

mà phải là E. Vậy E kề với cả ba đỉnh A,B,C. (Hình 4.10b)

46

Page 51: SÁU PHƯƠNG PHÁP GIẢI CÁC BÀI TOÁN PHỔ THÔNG

Chương 4. Phương pháp đồ thị

A B

C

A B

C

E

Hình 4.10a Hình 4.10b

Loại bỏ một trong ba đỉnh A,B,C khỏi bộ 4, chẳng hạn C và kết nạp đỉnhtùy ý F . Khi đó trong bộ 4 mới A,B,E, F lại có ít nhất một đỉnh kề với 3 đỉnhcòn lại. Nếu F là đỉnh kề với 3 đỉnh còn lại, thì nó phải kề với E, nên trong mọitrường hợp E vẫn là đỉnh kề với 3 đỉnh còn lại. (Hình 4.11)

A B

E F

Hình 4.11

Vì F là đỉnh tùy ý của đồ thị G và khác với các đỉnh A,B,C,E nên E kề vớitất cả các đỉnh còn lại của G. Bởi vậy, bậc của E bằng n− 1.

Đỉnh E lại được chọn một cách tùy ý trong các đỉnh của đồ thị G khác vớiA,B,C nên trong G chỉ có ba đỉnh A,B,C bậc nhỏ hơn n− 1.

Bước 3: Thuyết minh lời giảiTheo cách xác định cạnh của đồ thị G cặp đỉnh x, y kề nhau khi và chỉ khi

cặp số x, y có ước chung, mà trong G là có ít nhất n − 3 đỉnh kề với tất cả cácđỉnh còn lại. Do đó trong n số đã chọn ra có ít nhất n − 3 số có ước chung vớitất cả các số còn lại.

47

Page 52: SÁU PHƯƠNG PHÁP GIẢI CÁC BÀI TOÁN PHỔ THÔNG

Chương 4. Phương pháp đồ thị

Ví dụ 4.3.7. Một cơ quan cần tuyển ba người để thành lập một nhóm có đủnăng lực biên dịch các tài liệu từ sáu thứ tiếng: Anh, Pháp, Nga, Đức, TrungQuốc và Bồ Đào Nha sang tiếng Việt. Có bảy người đến dự tuyển, trong đó mỗingười đều biết hai và chỉ hai trong sáu thứ tiếng nói trên và bất cứ hai người nàocũng biết nhiều nhất một thứ tiếng chung trong sáu thứ tiếng đó. Biết rằng thứtiếng nào cũng có ít nhất hai người biết. Liệu có thể xảy ra trường hợp khôngtuyển chọn được như yêu cầu đã nêu hay không?

Lời giải: Ta sẽ chứng minh khẳng định đúng, tức là có thể tuyển chọn đượcnhư yêu cầu đã nêu.

1. Trước hết chứng minh khẳng địnhCho đơn đồ thị G có 6 đỉnh và 7 cạnh, bậc của mỗi đỉnh lớn hơn hoặc bằng

2. Khi đó trong G luôn tìm được 3 cạnh đôi một không kề nhau.a) Đồ thị G liên thông.Thật vậy, nếu G không liên thông thì do G gồm 6 đỉnh với bậc không bé hơn

2, phải chia thành 2 tam giác nên G chỉ có 6 cạnh. Ta đi tới mâu thuẫn với điềukiện đồ thị gồm 7 cạnh.

b) Đồ thị phải có chu trìnhThật vậy, nếu G không có chu trình và G lại liên thông nên G là một cây.

Khi đó G có đỉnh treo, tức là đỉnh bậc 1, nên mâu thuẫn với điều kiện bậc củamỗi đỉnh thuộc G đều không bé hơn 2.

c) Vì 7 × 2 = 14 = 6 × 2 + 2, nên G phải có 2 đỉnh bậc 3 hoặc 1 đỉnh bậc 4,các đỉnh còn lại bậc 2.

Ta xét từng trường hợp:Trường hợp a: G có một đỉnh bậc 4 (chẳng hạn đỉnh A). Không mất tính

tổng quát, ta giả sử 4 cạnh xuất phát từ A là AB,AD,AN,AT.

Lúc đó đỉnh P không thể kề với A, phải kề với 2 đỉnh khác, chẳng hạn P vàN và 2 đỉnh còn lại (B và D) phải kề nhau. Khi đó ba cạnh không kề nhau từngđôi một là BD,NT, PA. (Hình 4.12)

48

Page 53: SÁU PHƯƠNG PHÁP GIẢI CÁC BÀI TOÁN PHỔ THÔNG

Chương 4. Phương pháp đồ thị

A

B

D NP

T

Hình 4.12

Trường hợp b: G có hai đỉnh bậc 3. Giả sử A và P là hai đỉnh bậc 3. Vì Gliên thông, nên có ít nhất một đường đi từ A đến P . Ta tạm bỏ đường đó đi(vẫn giữ nguyên các đỉnh A và P ). Có thể xảy ra hai khả năng:

b1) G không còn liên thông. Như vậy, ta có hai chu trình sơ cấp phân biệt,mỗi chu trình gồm 3 cạnh. Suy ra ta chọn cạnh AP và hai cạnh không kề với nólà BT,ND. (Hình 4.13)

T

B

A

P

N

D

Hình 4.13

b2) G vẫn liên thông. Như vậy ta còn một chu trình sơ cấp (vì mỗi đỉnh đềucó bậc 2). Ta có 3 đường đi từ A đến P (một đường đã tạm bỏ, hai đường theochu trình), trong đó đường ngắn có độ dài 1 hoặc độ dài 2.

49

Page 54: SÁU PHƯƠNG PHÁP GIẢI CÁC BÀI TOÁN PHỔ THÔNG

Chương 4. Phương pháp đồ thị

N

A

B

T

P

DA

N B

P

T

DHình 4.14a Hình 4.14b

- Khi AP có độ dài 1, trên chu trình sơ cấp có độ dài 6 ta luôn chọn được bacạnh không kề nhau AD,PT,BN . (Hình 4.14a)

- Khi AP có độ dài 2, chỉ việc chọn một cạnh thuộc đường đi ngắn nhất từA đến P và hai cạnh không kề với nó trên chu trình sơ cấp còn lại, chẳng hạnAD, TP,NB hoặc DP,AT,NB. (Hình 4.14b)

Vậy trong mọi trường hợp ta đều tìm được ba cạnh đôi một không kề nhau.Khẳng định được chứng minh.2. Chuyển bài toán về dạng đồ thịDùng 6 điểm tương ứng với 6 ngoại ngữ nói trên. Dùng ngay chữ cái đầu của

tên ngoại ngữ để ghi tên các điểm tương ứng: A, P,N,D, T, B.Mỗi người biết hai ngoại ngữ nào đó được biểu thị bởi một cạnh nối hai đỉnh

tương ứng với hai ngoại ngữ đó.Đồ thị nhận được kí hiệu là G.Có 7 người tham gia dự tuyển và mỗi người biết hai ngoại ngữ, do đó đồ thị

G có 7 cạnh.Bất kỳ hai người dự tuyển nào cũng chỉ cùng biết tối đa một ngoại ngữ nói

trên, nên trong đồ thị G không có một cặp đỉnh nào được nối bằng hai cạnh.Vậy G là một đồ thị đơn gồm 6 đỉnh 7 cạnh. Do đó, theo khẳng định trên,

luôn luôn tìm được ba cạnh không kề nhau từng đôi một. Khi đó chỉ việc chọnba người được biểu thị bằng 3 cạnh nói trên, thì mỗi người này đều biết haingoại ngữ và không có hai người nào cùng biết một ngoại ngữ. Do đó nhóm bangười này biết tất cả sáu ngoại ngữ: Anh, Pháp, Nga, Đức, Trung Quốc và BồĐào Nha.

50

Page 55: SÁU PHƯƠNG PHÁP GIẢI CÁC BÀI TOÁN PHỔ THÔNG

Chương 4. Phương pháp đồ thị

Ví dụ 4.3.8. Mười bảy nhà khoa học đến dự hội nghị Quốc tế. Mỗi người trongsố họ chỉ biết một trong ba ngoại ngữ: Anh, Nga, Pháp. Chứng minh rằng có ítnhất 3 nhà khoa học cùng biết một trong ba ngoại ngữ nói trên.

Lời giải:1. Xây dựng đồ thị mô tả quan hệ.-) Đỉnh: Lấy 17 điểm trên mặt phẳng hay trong không gian tương ứng với 17

nhà khoa học. Dùng ngay tên các nhà khoa học để ghi trên các điểm tương ứng.-) Cạnh tô màu: Cặp điểm x, y được nối bằng đoạn thẳng hay đoạn cong tô

• Màu xanh khi và chỉ khi hai nhà khoa học tương ứng cùng biết tiếng Pháp.

• Màu vàng khi và chỉ khi hai nhà khoa học tương ứng cùng biết tiếng Anh.

• Màu đỏ khi và chỉ khi hai nhà khoa học tương ứng cùng biết tiếng Nga.

Đồ thị G nhận được mô tả toàn bộ hiện trạng về ngoại ngữ của các nhà khoahọc đến dự hội nghị.

2. Đáp án của bài toán bằng ngôn ngữ đồ thịĐồ thị G gồm 17 đỉnh, đầy đủ và các cạnh được tô bằng ba màu, nên theo

Định lý 4.1.5, với n = 3, có tam giác cùng màu, khi đó ba nhà khoa học tươngứng với các đỉnh của tam giác cùng biết một trong ba ngoại ngữ: Anh, Pháp,Nga.

Ví dụ 4.3.9. Một quần đảo có 2n(n ≥ 1) hòn đảo. Mỗi hòn đảo có đường ngầmnối trực tiếp với ít nhất n hòn đảo khác. Chứng minh rằng từ một hòn đảo bấtkỳ thuộc quần đảo đều có thể đi tới bất kỳ hòn đảo nào khác thuộc quần đảo nàybằng đường ngầm.

Lời giải:1. Xây dựng đồ thị mô tả các quan hệ

• Đỉnh: Lấy 2n điểm tương ứng với 2n hòn đảo. Dùng ngay tên các hòn đảođể ghi các điểm tương ứng.

• Cạnh: Cặp điểm x, y được nối bằng một đoạn thẳng hoặc một đoạn congkhông đi qua các điểm chung gian khác khi và chỉ khi hai hòn đảo tươngứng có đường ngầm nối với nhau.

51

Page 56: SÁU PHƯƠNG PHÁP GIẢI CÁC BÀI TOÁN PHỔ THÔNG

Chương 4. Phương pháp đồ thị

Đồ thị nhận được kí hiệu bằng G. Nó mô tả toàn bộ mối liên hệ bằng đườngngầm của quần đảo.

2. Đáp án của bài toán bằng "ngôn ngữ đồ thị"Đối với mỗi đỉnh x của đồ thị G số cạnh xuất phát từ nó bằng đúng số đường

ngầm xuất phát từ hòn đảo tương ứng. Bởi vậy bậc của mỗi đỉnh không nhỏhơn n, nên tổng bậc của hai đỉnh tùy ý không nhỏ hơn số đỉnh của đồ thị (2n).Khi đó, theo Định lý (4.1.1), đồ thị G liên thông. Do đó hai đỉnh tùy ý x, y củađồ thị G có đường nối với nhau. Mặt khác, mỗi cạnh của đường này biểu thịmột đường ngầm nối trực tiếp giữa hai hòn đảo, nên đường nối giữa hai đỉnhx, y biểu thị đường ngầm nối giữa hai hòn đảo tương ứng với x và y. Do đó từhòn đảo bất kỳ đều có thể đi bằng đường ngầm tới các hòn đảo còn lại của quầnđảo.

52

Page 57: SÁU PHƯƠNG PHÁP GIẢI CÁC BÀI TOÁN PHỔ THÔNG

Chương 5

Phương pháp bảng

5.1 Giới thiệu về phương pháp bảng

Nhiều bài toán phổ thông có thể giải bằng cách lập bảng mô tả mối quan hệgiữa các đối tượng được cho trong bài toán.

Đối với một số bài toán phổ thông trong đó xuất hiện hai hay nhiều đốitượng và các cặp phần tử nói lên mối quan hệ giữa các tệp người ta có thể thiếtlập một hay nhiều bảng, để mô tả mối quan hệ giữa các tệp.

Mỗi bảng này có hàng trên cùng ghi các phần tử của một tệp, còn cột tậncùng bên trái ghi các phần tử thuộc tập kia và các vị trí trong bảng ghi mã sốquan hệ giữa các phần tử thuộc các tệp.

Căn cứ vào các điều kiện đã cho trong bài toán gạch bỏ đi những cặp phầntử không thích hợp từ đó đi đến lời giải của bài toán.

5.2 Một số ví dụ minh họa

Ví dụ 5.2.1. Trong buổi học nữ công ba bạn Cúc, Đào, Hồng làm ba bông hoa:cúc, đào, hồng. Bạn làm hoa hồng nói với bạn Cúc "Thế là trong chúng ta khôngcó ai làm loại hoa trùng với tên mình". Hãy xác định tên hoa mà mỗi bạn đãlàm?

Lời giải:Bài toán này có hai tệp đối tượng. Tệp thứ nhất gồm các bạn làm hoa, tệp

thứ hai gồm các bông hoa được làm. Ta có thể giải bằng phương pháp bảng nhưsau1. Lập bảng

53

Page 58: SÁU PHƯƠNG PHÁP GIẢI CÁC BÀI TOÁN PHỔ THÔNG

Chương 5. Phương pháp bảng

Bảng cần lập gồm 4 hàng và 4 cột. Hàng đầu, từ cột thứ hai ghi lần lượt têncác bông hoa được làm viết tắt là các chữ cái đầu, còn trên cột tận cùng bêntrái từ hàng hai ghi lần lượt tên các bạn tham gia làm hoa viết tắt là chữ cáiđầu viết hoa.2. Điền mã số quan hệ vào các vị trí của bảng

a) Căn cứ vào giả thiết: Mỗi bạn đều không làm hoa trùng với tên mình, màđiền mã "k" vào các ô nằm trên đường chéo chính.

Nguoic d h

hoa

C

D

H

kk

k

k

Bảng 5.1

b) Căn cứ vào câu "Bạn làm hoa hồng nói với bạn Cúc" suy ra bạn Cúckhông phải làm hoa hồng, mà ghi mã "k" vào ô nằm ở hàng Cúc, cột hồng.3. Loại bỏ vị trí không thỏa mãn quan hệ để nhận được lời giải

Trong bảng trên cột cuối vị trí 1 và 3 bị gạch bỏ, nên vị trí duy nhất còn lạilà vị trí thứ hai phải thỏa mãn quan hệ giữa người làm hoa và hoa được làm.Do đó bạn Đào làm hoa hồng.

Vì trên hàng 2 Đào đã có vị trí thỏa mãn quan hệ nên toàn bộ hàng này bịloại ra khỏi diện xét. Bởi vậy cột Cúc chỉ còn vị trí cuối cùng trong diện xét.Bởi vậy nó phải thỏa mãn quan hệ giữa người làm hoa và hoa được làm, nênbạn Hồng làm hoa cúc.

Từ đó suy ra người còn lại bạn Cúc phải làm hoa đào.Vậy Bạn Cúc làm hoa đào, Bạn Đào làm hoa hồng, Bạn Hồng làm hoa cúc.

Ví dụ 5.2.2. Ba bạn Long, Hoan, Lan đều là học sinh giỏi toán. Cô giáo thưởngcho ba bạn thứ đồ chơi là: Búp bê, ô tô và con quay điện. Biết rằng bạn Longkhông thích chơi búp bê, bạn Hoan không nhận búp bê và cũng không thích ô tô.

Hỏi cô giáo đã thưởng cho ai đồ chơi gì?

Lời giải: Bài toán gồm hai nhóm đối tượng:

54

Page 59: SÁU PHƯƠNG PHÁP GIẢI CÁC BÀI TOÁN PHỔ THÔNG

Chương 5. Phương pháp bảng

• Nhóm thứ nhất gồm các bạn: Long, Hoan, Lan.

• Nhóm thứ hai là các đồ chơi mà cô giáo thưởng cho các bạn: Búp bê, ô tô,con quay.

Ta lập một bảng có hàng trên là tên các bạn, và cột bên trái là các đồ chơi.

Long Hoan Lan

Búp bê 0 0 ×

Ô tô × 0

Con quay ×

Bảng 5.2

Nhìn vào bảng, ta suy luận và có kết quả sau:Lan được thưởng con búp bê.Long được thưởng ô tô.Hoan được thưởng con quay.

Ví dụ 5.2.3. Có sáu em học sinh: An, Bình, Nam, Long, Trang, Minh thamgia cuộc thi leo núi. Có hai em về được đích, đáp lại câu hỏi ai về tới đích, cónăm câu trả lời:

1. An và Nam.

2. Long và Trang.

3. Minh và Long.

4. An và Minh.

5. An và Linh.

Thật ra trong năm câu trả lời trên có 4 câu mà mỗi câu chỉ đúng một nửa(đúng một tên), còn sai một nửa, còn một câu sai cả hai tên. Hỏi ai đã về tớiđích của cuộc thi?

Lời giải: Dựa vào năm câu trả lời, ta lập bảng sau:

55

Page 60: SÁU PHƯƠNG PHÁP GIẢI CÁC BÀI TOÁN PHỔ THÔNG

Chương 5. Phương pháp bảng

An Long Nam Linh Trang Minh

1 1 1

2 1 1

3 1 1

4 1 1

5 1 1

Bảng 5.3

Dựa vào Bảng 5.3 để suy luận ra đáp án:Dùng số 1 để chỉ khẳng định: "Đối tượng về được đích". Bởi vậy trong Bảng

5.3 tại vị trí giao của cột đối tượng về được đích và hàng tương ứng với câu trảlời có tên đối tượng này đều được ghi số 1.

Theo giả thiết, có 4 câu, mà mỗi câu có 1 ý đúng, nên bốn hàng tương ứngvới 4 câu này đều có đúng một số 1. Một câu trả lời sai cả hai ý, nên hàng tươngứng với câu này không có số 1.

Như vậy, đáp án là cặp đối tượng mà trên Bảng 5.3 sau ghi số 1 chỉ đối tượngvề đích có bốn hàng ghi một số 1, một hàng không có số 1 nào.

Ta suy luận xuất phát từ các đối tượng theo thứ tự từ trái sang phải, đồngthời kết hợp thành cặp cũng lần lượt theo thứ tự từ trái sang phải.

1) Nếu An về được đích, thì tại cột 1 của các hàng 1, 4, 5 được ghi số 1.

a) Nếu Long về tới đích cùng với An, thì thêm cột 2 của hàng 2 và hàng3 được ghi số 1.

Như vậy, cả 5 hàng đều có số 1, nên Long không thể kết hợp với Anđể về tới đích, hay Long bị loại.

b) Nếu Nam về tới đích cùng với An, thì cột 3 của hàng 1 được ghi số 1.Như vậy hàng 1 có hai số 1, nên Nam bị loại.

c) Nếu Linh về tới đích cùng với An, thì tại cột 3 của hàng 5 được ghi số1.

Như vậy, hàng 5 sẽ có hai số 1, nên Linh bị loại.

d) Nếu Trang về tới đích cùng với An, thi tại cột 5 hàng 2 được ghi số 1.

Như vậy, bốn hàng 1, 2, 4, 5 đều có một số 1, còn hàng 3 không có số 1

nào, nên Trang được chấp nhận về tới đích cùng với An.

e) Nếu Minh về tới đích cùng với An, thì tại cột 6 của hàng 4, hàng 5 đượcghi số 1.

56

Page 61: SÁU PHƯƠNG PHÁP GIẢI CÁC BÀI TOÁN PHỔ THÔNG

Chương 5. Phương pháp bảng

Như vậy, hàng 4 có hai số 1, nên Minh bị loại.

2) Nếu Long về tới đích, thì tại cột 2 hàng 2 và hàng 3 được ghi số 1.

a) Nếu Nam về tới đích cùng với Long, thì thêm cột 3 hàng 1 được ghi số1. Như vậy, tại hàng 1 có hai số 1, nên Nam bị loại.

b) Nếu Linh về tới đích cùng với Long, thì thêm cột 4 hàng 5 được ghi số1. Như vậy, tại hàng 5 có hai số 1, nên Linh bị loại.

c) Nếu Trang về tới đích cùng với Long, thì thêm cột 6 hàng 2 được ghisố 1. Như vậy, tại hàng 2 có hai số 1, nên Trang bị loại.

d) Nếu Minh về tới đích cùng với Long, thì thêm cột 7 hàng 3 và hàng 4

được ghi số 1. Như vậy, hàng 1 và hàng 5 không có số 1, nên Minh bịloại.

Như vậy, nếu Long về tới đích, thì chỉ có một người về tới đích, nênLong bị loại.

3) Nếu Nam về tới đích, thì tại cột 3 hàng 1 ghi số 1.

a) Nếu Linh về tới đích cùng với Nam, thì thêm cột 4 hàng 5 được ghi số1. Như vậy, ba hàng 2, 3, 4 không có số 1 nên Linh bị loại.

b) Nếu Trang về tới đích cùng với Nam, thì thêm cột 5 hàng 2 được ghisố 1. Như vậy, ba hàng 3, 4, 5 không có số 1, nên Trang bị loại.

c) Nếu Minh về tới đích cùng với Nam, thì thêm cột 6 hàng 3, 4 được ghisố 1. Như vậy, hàng 2, 5 không có số 1, nên Minh bị loại.

Như vậy, nếu Nam về tới đích, thì chỉ về một mình, nên Nam bị loại.

4) Nếu Linh về tới đích, thì cột 4 hàng 5 được ghi số 1.

a) Nếu Trang về tới đích cùng với Linh, thì thêm cột 5 hàng 2 được ghi số1. Như vậy, ba hàng 1, 4, 5 không có số 1, nên Trang bị loại.

b) Nếu Minh về tới đích cùn với Linh, thì thêm cột 6 hàng 3, 4 được ghisố 1. Như vậy, hai hàng 1 và hàng 5 không có số 1, nên Minh bị loại.

Nếu Linh về tới đích, thì chỉ về một mình, nên Linh bị loại.

5) Nếu Trang về tới đích, thì tại cột 5 hàng 2 được ghi số 1.

Nếu Minh về tới đích cùng với Trang, thì thêm cột 6 hàng 3, 4 được ghi số1. Như vậy, hai hàng 1, 5 không có số 1, nên Minh bị loại.

Tại phần 1, Trang đã kết hợp được với An để về đích.

57

Page 62: SÁU PHƯƠNG PHÁP GIẢI CÁC BÀI TOÁN PHỔ THÔNG

Chương 5. Phương pháp bảng

6) Nếu Minh về tới đích, thì chỉ về một mình, nên Minh bị loại.

Từ quá trình lý luận ở trên, suy ra cặp đối tượng duy nhất về tới đích. Đólà em An và em Trang.

Ví dụ 5.2.4. Bốn bạn Thu, Hoa, Hoan, Trang nhận được điểm của bài kiểm tracuối học kỳ. Bạn Lan cùng lớp muốn biết điểm của từng người. Khi Lan hỏi thìđược các bạn úp mở trả lời như sau:

Thu nói: "Bạn Trang được 7, Hoa được 8, Hoan được 9"Hoa nói: "Bạn Trang được 10, Hoan được 8, Thu được 9"Hoan nói: "Cả ba bạn đều được 7"Trang nói: "Cả ba bạn đều được 8"Biết rằng không có bạn nào được ba bạn nói cùng đúng với số điểm của mình

và mỗi câu trả lời ở trên chỉ nói đúng số điểm của một người mà thôi. Hãy tìmđiểm của mỗi người?

Lời giải: Ta ghi điểm của từng người trong các câu trả lời vào trong bảng sau.Mỗi hàng là câu trả lời của bạn. Mỗi cột là điểm của một bạn mà các bạn trảlời.

Thu Hoa Hoan Trang

Thu 8 9 7

Hoa 9 8 10

Hoan 7 7 7

Trang 8 8 8

Bảng 5.4

Dựa vào bảng 5.4 đồng thời căn cứ vào điều kiện: Không có bạn nào đượchai bạn nói đúng điểm của mình ta suy ra:

Hoa không được điểm 8.Hoan không được điểm 8.Trang không được điểm 7

Căn cứ vào điều kiện: Mỗi câu trả lời chỉ đúng với số điểm của một người tasuy ra: Thu được 8, Hoa được 7, Hoan được 9 và Trang được 10.

Ví dụ 5.2.5. Lớp em có 35 học sinh, trong đó có 20 bạn trai. Chủ nhật vừa quacó 8 bạn gái đi xem phim và có 11 bạn trai không đi xem. Hỏi đã có bao nhiêubạn không đi xem phim?

58

Page 63: SÁU PHƯƠNG PHÁP GIẢI CÁC BÀI TOÁN PHỔ THÔNG

Chương 5. Phương pháp bảng

Lời giải:Trong bài toán này ta xây dựng hai nhóm đối tượng là:• Nhóm 1: Nam, Nữ, Tất cả.• Nhóm 2: Có xem phim, Không xem phim, Tất cả.Ta lập một bảng có hàng trên cùng là nhóm 1 và cột bên trái là nhóm 2 như

sau:

Nam Nữ Tất cả

Có xem phim 9 8 17

Không xem phim 11 7 18

Tất cả 20 15 35

Bảng 5.5

Dựa vào bảng trên, ta có:Số bạn nam có đi xem là:

20− 11 = 9(bạn)

Suy ra: Điền 9 vào ô (Có - Nam).Số bạn đi xem phim là:

9 + 8 = 17(bạn)

Suy ra: Điền 17 vào ô (Có - Tất cả)Số bạn không đi xem là:

35− 17 = 18(bạn)

Suy ra: Điền 18 vào ô (Không - Tất cả)Vậy có 18 bạn không đi xem phim.

Ví dụ 5.2.6. Trong một buồng trên toa tàu có 6 hành khách A,B, V,G,D,E từ6 thành phố M,L,K, T,X,O. Dọc đường họ phát hiện ra:

1) A và một người từ M là bác sĩ, D và một người từ L là giáo viên, V vàmột người từ T là kĩ sư.

2) B,E và một người từ K là cựu chiến binh, còn người từ T chưa nhập ngũbao giờ.

59

Page 64: SÁU PHƯƠNG PHÁP GIẢI CÁC BÀI TOÁN PHỔ THÔNG

Chương 5. Phương pháp bảng

3) Người từ X già hơn A, người từ O già hơn V , còn E là người trẻ nhất hội.4) B và người từ M đi K, còn V và người từ X đi O.

Hãy xác định xem ai là người thành phố nào và nghề nghiệp của từng người?

Lời giải:Trước tiên ta phải xác định xem ai là người thành phố nào bằng cách lập

bảng hai chiều: chiều thẳng đứng ghi tên người, chiều ngang ghi tên thành phố.Bảng này chứa tất cả các khả năng có thể để kết hợp tên người với tên

thành phố. Tức là một tích đế các của hai tập: H = {A,B, V,G,D,E}; P =

{M,L,K, T,X,O}.Quy ước rằng ô bị gạch bỏ theo điều kiện i sẽ ghi số i trên đó (1 ≤ i ≤ 4).Trong đề bài có nói A và người từ M là bác sĩ ⇒ A không là người từ M ⇒

ô (A,M) trong bảng phải bị gạch bỏ (ta sẽ ghi số 1 vào ô này.)Tiếp tục điều kiện 1 người từ L là giáo viên, từ T là kĩ sư suy ra A không thể

là người từ L, T (vì A: bác sĩ) ⇒ gạch nốt (A,L), (A, T ).Hoàn toàn tương tự với D ta thấy anh ta không phải từ L, từ M , từ

T và V không từ T, L,M nên trong bảng các ô (D,M), (D,L), (D, T ) và(V,M), (V, L), (V, T ) phải đánh số 1 (Bảng 5.6).

M L K T X O

A

B

V

G

D

E

1 1 1

1 1 1

1 1 1

M L K T X O

A

B

V

G

D

E

1 1 1

1 1 1

1 1 1

2 2

22

Bảng 5.6 Bảng 5.7

Từ điều kiện số 2 suy ra B và E không phải từ T , nên các ô (B, T ), (E, T )

phải ghi số 2. Ngoài ra B và E cũng không từ K nên (B,K) và (E,K) cũng phảighi số 2 (Bảng 5.7).

60

Page 65: SÁU PHƯƠNG PHÁP GIẢI CÁC BÀI TOÁN PHỔ THÔNG

Chương 5. Phương pháp bảng

Trên Bảng 5.7 ta nhận thấy cột T chỉ còn một ô trống (G, T ) tức là G làngười từ T . Bởi vậy tại ô (G, T ) đặt dấu ∗ và gạch bỏ những ô còn lại của hàngG (Bảng 5.8).

M L K T X O

A

B

V

G

D

E

1 1 1

1 1 1

1 1 1

M L K T X O

A

B

V

G

D

E

1 1 1

1 1 1

1 1 1

2 2

22

2 2

22

3

3

4

4

33

4

Bảng 5.8 Bảng 5.9

Từ điều kiện 3 suy ra A và E không từ X, nên đánh số 3 vào các ô (E,X) và(A,X), tương tự các ô (V,O) và (E,O) cũng được ghi số 3 (Bảng 5.9).

Từ điều kiện 4 suy ra B không từ M và X, còn V cũng không từ X. Bởi vậycần gạch bỏ (B,X), (B,M), (V,X) (ghi số 4 vào các ô này) (Bảng 5.9).

Trong hàng V cũng như trong các cột M,X đều còn duy nhất một ô, nên V

là người từ K, E là người từ M , D là người từ X. Bởi vậy ta đặt dấu ∗ vào cácô (V,K), (E,M) và (O,X) đồng thời gạch những ô còn lại, trên cột qua (V,K),trên các hàng qua (D,X) và (E,M) (Bảng 5.10).

Trong Bảng 5.10 hàng A chỉ còn duy nhất ô (A,O) là trống, nên A là ngườitừ O. Bởi vậy tại ô (A,O) đặt ∗ và gạch ô trống còn lại trên cột O là (B,O).Khi đó trên hàng B còn duy nhất một ô trống là (B,L), nên B là người từ L vàô (B,L) được ghi ∗ (Bảng 5.11).

61

Page 66: SÁU PHƯƠNG PHÁP GIẢI CÁC BÀI TOÁN PHỔ THÔNG

Chương 5. Phương pháp bảng

M L K T X O

A

B

V

G

D

E

1 1 1

1 1 1

1 1 1

M L K T X O

A

B

V

G

D

E

1 1 1

1 1 1

1 1 1

2 2

22

2 2

22

3

3

4

4

33

4

∗ ∗

∗4 4

4

3

3

33

Bảng 5.10 Bảng 5.11

Căn cứ vào điều kiện đã cho từ Bảng 5.11 suy ra nghề nghiệp của từng ngườinhư sau: A và E là bác sĩ, D và B là giáo viên, V và G là kĩ sư. Và kết luậnđược: Bác sĩ A người ở O, giáo viên B người ở L, giáo viên D người ở X, bác sĩE người ở M , kĩ sư G người ở T và kĩ sư V người ở K.

Ví dụ 5.2.7. Có ba em bé từ Quy Nhơn, Ninh Bình và Huế tới dự trại hè. Chúngkhác tuổi nhau và cùng thích thể thao. Chỉ có Long và em bé từ Quy Nhơn chơitennis, Loan và em bé từ Ninh Bình chơi bóng, Dung chơi cờ tướng và lớn hơnem bé từ Ninh Bình. Các em bé chơi tennis không chơi cờ. Em bé chơi cờ lớnnhất.

Hỏi em bé nào đến từ thành phố nào và thích môn thể thao nào? Ai lớn hơnai?

Lời giải: Trước tiên ta xác định xem ai là người thành phố nào? Thích môn thểthao nào?

Ta lập bảng hai chiều: Chiều thẳng đứng ghi tên môn thể thao và thành phố,chiều ngang ghi tên các em bé và thành phố. Bảng này chứa tất cả các khả năngcó thể để kết hợp tên người và tên môn thể thao ưa thích, tên người với tênthành phố, tức là những tích đề các của hai tập hợp: A×X và A× Y với

A = {Long,Loan,Dung}X = {T,B, C}Y = {Q,N,H}

Từ giả thiết ta có:

62

Page 67: SÁU PHƯƠNG PHÁP GIẢI CÁC BÀI TOÁN PHỔ THÔNG

Chương 5. Phương pháp bảng

Do Long và em đến từ Q chơi tennis, suy ra Long không ở Q, vì vậy ô(Q,Long) ghi 0.

Do Loan và em đến từ N chơi bóng đá, suy ra Loan không ở N, vì vậy ô(N,Loan) ghi 0

Suy ra Dung không ở Q vì em bé từ Q chơi Tennis (mà chơi tennis thì khôngchơi cờ), do đó Dung ở H, vì vậy ô (H, Dung) đánh dấu ×.

Suy ra các ô còn lại ở dòng H gạch bỏ và các ô (Q,Dung), (N,Dung) cũnggạch bỏ.

Dòng Q còn lại duy nhất ô (Q,Loan) trống, nên ô này đánh dấu ×.Dòng N còn lại duy nhất ô (N,Long) trống, nên ô này ghi dấu ×.Cũng từ giả thiết ta có:

các ô (T, Long); (T,Q) ghi dấu ×các ô (B,Loan); (B,N) ghi dấu ×ô (C,Dung) ghi dấu ×.

Vậy: Long chơi tennis và đến từ Ninh Bình.Loan chơi bóng và đến từ Quy Nhơn.Dung chơi cờ và đến từ Huế.

Long Loan Dung

T

B

C

Q

N

H

×

NQ H

××

××

×

×

0

0

0 0

0

Bảng 5.12

63

Page 68: SÁU PHƯƠNG PHÁP GIẢI CÁC BÀI TOÁN PHỔ THÔNG

Chương 5. Phương pháp bảng

Long

Loan

Dung

T B C Q N H

×

××

×

×

×

Bảng 5.13

Ví dụ 5.2.8. Trong một nhóm 4 sinh viên có tên là B, V,N, P với họ là O,K,Z,H

và học tại 4 khóa 1, 2, 3, 4 cùng trường. Hãy xác định họ và tên năm học của từngsinh viên biết rằng:

(i) B là sinh viên được học bổng toàn phần.

(ii) V hè tới phải đi thực tập, còn sinh viên có họ H được nghỉ hè về thăm giađình.

(iii) N học trước P một khóa.

(iv) B và sinh viên có họ O là người cùng quê.

(v) Sinh viên họ K năm ngoái tốt nghiệp phổ thông trung học và thi đỗ vàokhoa mà sinh viên có họ Z đang học.

(vi) B đôi khi sử dụng vở ghi năm ngoái của V .

Lời giải: Lập bảng hai chiều. Chiều thẳng đứng ghi tên sinh viên và 4 khóahọc: B, V,N, P, I, II, III, IV . Chiều nằm ngang ghi họ sinh viên và 4 khóa học:O,K,Z,H, I, II, III, IV . Bảng này thể hiện đầy đủ tất cả các quan hệ có thể cógiữa tên và họ, tên và năm học, họ và năm học.

64

Page 69: SÁU PHƯƠNG PHÁP GIẢI CÁC BÀI TOÁN PHỔ THÔNG

Chương 5. Phương pháp bảng

O K Z H I

II

III IV

B

V

N

P

I

II

III

IV

4 4

*

* *

*

*

*

*

*

*

*

*

*

1

2

Bảng 5.14

Vì học bổng toàn phần không bao giờ cấp cho sinh viên năm thứ nhất, nêntừ điều kiện 1 suy ra B không học năm thứ nhất, nên ô (B, I) ghi số 1.

Vì N học hơn P một khóa và V có vở cũ (điều kiện 6), nên chỉ P là sinh viênnăm thứ nhất. Ở bảng ta ghi ô (P, I) bằng một dấu ∗.

Vì B dùng vở năm ngoái của V , nên B học năm thứ III và V học năm thứIV . Bởi vậy các ô (B, III) và (V, IV ) được ghi dấu ∗. Theo điều kiện 5, sinh viênhọ K học năm thứ nhất, nên ô (I,K) được ghi dấu ∗. Nhưng ta đã biết P họcnăm thứ nhất nên P có họ K, vì vậy ô (P,K) được ghi dấu ∗. Do đó ta gạch bỏnhững ô còn lại của hàng P và cột K.

Từ điều kiện 2 suy ra V không mang họ H, nên tại ô (V,H) đặt số 2.Từ điều kiện 4 suy ra B không mang họ O, nên tại ô (B,O) đặt số 4.Kết hợp điều kiện 2 và điều kiện 4 suy ra B không mang họ H vì B ở cùng

quê với sinh viên có họ O, còn sinh viên họ H về quê thăm gia đình, nên tại ô(B,H) đặt số 4.

Nhận thấy hàng B chỉ còn duy nhất một ô trống là (B,Z) suy ra sinh viênB mang họ Z, ta ghi vào ô (B,Z) dấu ∗. Trên cột H chỉ còn ô (N,H) trống nênN mang họ H, ta ghi dấu ∗ vào ô (N,H). Trên hàng N ta gạch bỏ những ô cònlại. Trên cột Z ta gạch bỏ những ô còn lại. Nhìn vào hàng V chỉ còn duy nhấtmột ô trống là (V,O), nên V mang họ O và ghi vào ô (V,O) dấu ∗.

Kết luận: Sinh viên B có họ Z học năm thứ III.

65

Page 70: SÁU PHƯƠNG PHÁP GIẢI CÁC BÀI TOÁN PHỔ THÔNG

Chương 5. Phương pháp bảng

Sinh viên V có họ O học năm thứ IV .Sinh viên N có họ H học năm thứ II.Sinh viên P có họ K học năm thứ I.

66

Page 71: SÁU PHƯƠNG PHÁP GIẢI CÁC BÀI TOÁN PHỔ THÔNG

Chương 6

Phương pháp sơ đồ

Đây là phương pháp gần tương tự như phương pháp bảng, song phương phápnày lợi thế hơn khi giải quyết các bài toán mà ở đó số tệp lớn hơn 2.

6.1 Các bước thực hiện phương pháp sơ đồ

6.1.1 Thiết lập sơ đồ

Lấy các nhóm điểm trên mặt phẳng hay trong không gian tương ứng với cáctệp. Dùng ngay kí hiệu các đối tượng để ghi trên các điểm tương ứng.

Mỗi cặp điểm tương ứng với hai đối tượng có một quan hệ nào đó đã chotrong bài toán được nối với nhau bằng một đoạn thẳng hoặc một đoạn cong đặctrưng cho quan hệ mà nó biểu thị và không đi qua các điểm tương ứng chunggian khác. Ta gọi sơ đồ nhận được là sơ đồ mô tả quan hệ.

6.1.2 Dựa vào cấu trúc của sơ đồ mô tả quan hệ và điều kiệnđã cho trong bài toán mà suy ra đáp án

6.2 Một số ví dụ

Ví dụ 6.2.1. Trong buổi học nữ công ba bạn Cúc, Đào, Hồng làm ba bông hoa:Cúc, đào, hồng. Bạn làm hoa hồng nói với bạn Cúc "Thế là trong chúng ta khôngcó ai làm loại hoa trùng với tên mình". Hãy xác định tên hoa mà mỗi bạn đãlàm?

Bài toán này đã được trình bày bằng phương pháp bảng. Dưới đây trình bàyquá trình giải bài toán trên bằng phương pháp sơ đồ.Lời giải:

67

Page 72: SÁU PHƯƠNG PHÁP GIẢI CÁC BÀI TOÁN PHỔ THÔNG

Chương 6. Phương pháp sơ đồ

1. Lập sơ đồTrong bài toán có hai nhóm đối tượng:

• Nhóm 1 gồm ba bạn Cúc, Đào, Hồng kí hiệu bằng ba điểm C,D,H.

• Nhóm 2 gồm ba bông hoa cúc, đào, hồng kí hiệu bằng ba điểm c, d, h.

Mối quan hệ của hai nhóm đối tượng này được kí hiệu bằng:

• Nét đứt nếu quan hệ giữa chúng là sai.

• Nét liền nếu quan hệ giữa chúng là đúng.

C

D

H

c

d

h

Hình 6.1:

Theo giả thiết bạn làm hoa hồng nói với bạn Cúc suy ra Cúc không làm hoahồng, nên C − h được nối nét đứt.

Theo giả thiết "chẳng có ai làm loại hoa trùng tên với mình" suy ra C −c,D − d,H − h được nối bằng nét đứt. Ta thấy C − c, C − h nối nét đứt suy raC − d nối nét liền. C − h, H − h nối nét đứt, do đó D − h và H − c nối nét liền.

Kết luận: Bạn Cúc làm hoa đào.Bạn Đào làm hoa hồng.Bạn Hồng làm hoa cúc.

Ví dụ 6.2.2. Ba bạn Thủy, Ánh, Hường mặc ba màu áo: Trắng, xanh, hồng vàcó ba dây buộc tóc cùng màu ấy. Biết rằng chỉ có Thủy là có màu áo và màu dâybuộc tóc trùng nhau, còn áo và dây buộc tóc của Ánh đều không là màu trắng.Hường có dây buộc tóc màu xanh. Hãy xác định màu áo và màu dây buộc tóccủa từng bạn?

68

Page 73: SÁU PHƯƠNG PHÁP GIẢI CÁC BÀI TOÁN PHỔ THÔNG

Chương 6. Phương pháp sơ đồ

Lời giải: Trong bài toán này có ba nhóm đối tượng:Nhóm 1 gồm ba bạn: Thủy, Ánh, Hường. Ta kí hiệu bằng ba điểm T,A,H.Nhóm 2 gồm ba màu áo: Trắng, xanh, hồng. Ta kí hiệu bằng ba điểm t0, x0, h0.Nhóm 3 gồm ba dây buộc tóc màu: Trắng, xanh, hồng. Ta kí hiệu bằng ba

điểm t, x, h.Mối quan hệ giữa các đối tượng của ba nhóm này được kí hiệu bằng:

• Nét đứt nếu quan hệ giữa chúng là sai.

• Nét liền nếu quan hệ của chúng là đúng.

T A H

t0

x0

h0

t

x

h

Hình 6.2

Theo giả thiết, áo và dây buộc tóc của Ánh đều không phải màu trắng, nênA− t0 và A − t được nối bằng nét đứt. Còn Hường có dây buộc tóc màu xanh,nên H−x được nối bằng nét liền. Suy ra chỉ còn T − t nối bằng nét liền. Vì Thủycó màu áo và dây buộc tóc trùng màu nhau, nên T − t0 cũng được nối bằng nétliền. Hơn nữa, theo giả thiết thì Ánh và Hường đều có áo và dây buộc tóc khácmàu, mà A − h được nối bằng nét liền, thì A − x0 cũng phải nối nét liền. Cuốicùng H − h0 cũng được nối bằng nét liền.

Vậy: Thủy mặc áo trắng và dây buộc tóc trắng.

69

Page 74: SÁU PHƯƠNG PHÁP GIẢI CÁC BÀI TOÁN PHỔ THÔNG

Chương 6. Phương pháp sơ đồ

Ánh mặc áo xanh và dây buộc tóc hồng.Hường mặc áo hồng và dây buộc tóc xanh.

Ví dụ 6.2.3. Trong một trường phổ thông cơ sở ở Bình Định có ba thầy giáolà Minh, Bình, Vinh dạy các môn Sinh vật, Lý, Toán, Hóa, tiếng Anh và tiếngPháp. Mỗi thầy dạy hai môn. Người ta biết về các thầy như sau:

• Thầy dạy Lý và thầy dạy tiếng Pháp là láng giềng của nhau.

• Thầy Minh trẻ nhất trong ba thầy.

• Thầy Bình, thầy dạy Sinh vật và thầy dạy tiếng Pháp thường đi với nhautrên đường về.

• Thầy dạy Sinh vật nhiều tuổi hơn thầy dạy môn Toán.

• Thầy dạy tiếng Anh, thầy dạy Toán và thầy Minh khi rảnh rỗi thường hayđánh quần vợt với một thầy thứ tư.

Hãy xác định xem mỗi thầy dạy hai môn nào?

Lời giải: Bài toán trên có hai nhóm đối tượng

• Nhóm thứ nhất gồm ba thầy giáo: Minh, Bình, Vinh.

• Nhóm thứ hai gồm sáu môn học tiếng Anh, tiếng Pháp, Toán, Sinh vật,Lý, Hóa.

Ta lấy chữ cái đầu của mỗi nhóm đối tượng trên để kí hiệu cho các điểmtương ứng.

Từ các điều kiện của bài toán ta có sơ đồ sau:

70

Page 75: SÁU PHƯƠNG PHÁP GIẢI CÁC BÀI TOÁN PHỔ THÔNG

Chương 6. Phương pháp sơ đồ

M

B

V

A

T

S

P

L

H

Hình 6.3

Ta có Minh - Toán được nối bằng nét đứt, nghĩa là thầy Minh không dạymôn Toán. Và Bình - Sinh vật được nối bằng nét đứt, nghĩa là thầy Bình khôngdạy môn Sinh vật. Từ (2) và (4) suy ra thầy Minh không dạy Sinh vật.

Còn từ các điều kiện khác trực tiếp cho ta các đường nét đứt theo sơ đồ.Theo sơ đồ trên, ta thấy thầy Minh chỉ có thể dạy hai trong ba môn Pháp,

Lý, Hóa. Nhưng thầy dạy Pháp và thầy dạy Lý là khác nhau nên thầy Minhhoặc là dạy Pháp - Hóa, hoặc là dạy Lý - Hóa. Nếu thầy Minh dạy Lý - Hoá,suy ra thầy Bình phải dạy Toán - Anh, nhưng điều đó không thể có vì thầy dạyToán và thầy dạy Anh là hai thầy khác nhau. Vậy thầy Minh dạy Pháp - Hóa.

Tiếp theo thầy Bình chỉ có thể dạy hai trong ba môn Lý, Toán, Anh nhưngToán, Anh không thể do một người dạy, nên thầy Bình hoặc là dạy Lý - Toánhoặc là dạy Lý - Anh. Nếu thầy Bình dạy Lý - Anh suy ra thầy Vinh dạy Toán- Sinh vật. Điều này không thỏa mãn vì thầy dạy Toán và thầy dạy Sinh vật làhai thầy khác nhau. Vậy thầy Bình dạy Lý - Toán, suy ra thầy Vinh dạy Sinhvật - Anh.

Vậy: Thầy Minh dạy hai môn Pháp - Hóa.Thầy Bình dạy hai môn Lý - Toán.Thầy Vinh dạy hai môn Sinh vật - Anh.

Ví dụ 6.2.4. Bốn bạn Dung, Anh, Linh, Trang nhận được điểm bài kiểm tramôn toán. Bạn Nam cùng lớp muốn biết điểm của từng người. Khi Nam hỏi thìđược các bạn trả lời úp mở như sau:

71

Page 76: SÁU PHƯƠNG PHÁP GIẢI CÁC BÀI TOÁN PHỔ THÔNG

Chương 6. Phương pháp sơ đồ

Dung nói: "Bạn Anh 7 điểm, bạn Linh 9 điểm, bạn Trang 8 điểm."Anh nói: "Bạn Trang 10 điểm, bạn Dung 8 và bạn Linh 7".Linh nói: "Cả ba bạn đều được 8".Trang nói: "Cả ba bạn đều được 7".Hãy xác định điểm của từng người? Biết rằng không có bạn nào được hai bạn

cùng nói đúng điểm của mình.

Lời giải: Lập sơ đồ: Lấy hai nhóm điểm tương ứng với hai nhóm đối tượng: Cácbạn học sinh và điểm số.

Điểm x được nối với điểm y bằng t đoạn thẳng song song khi và chỉ khi có t

bạn nói điểm của bạn x bằng số y.Như vậy, ta có sơ đồ sau:

Dung

Anh

Linh

Trang

10

9

8

7

Hình 6.4

Căn cứ vào điều kiện đã cho trong bài toán, cặp đỉnh được nối bằng từ 2

đoạn thẳng trở lên đều bị loạiDo đó Trang được điểm 10, Linh được 9, Anh được 8 và Dung được 7.

Ví dụ 6.2.5. Cho ba ngôi nhà, một cái giếng, một cái lán và một cái hầm chứađồ đạc. Cần phải vạch từ mỗi ngôi nhà một con đường tới giếng, một con đườngtới lán và một con đường tới hầm chứa đồ đạc, sao cho không có một con đườngnào trong chín con đường đó cắt một con đường khác. Bạn hãy chứng minh rằngkhông thể làm được việc đó.

Lời giải: Kí hiệu ba ngôi nhà lần lượt bằng A,B,C. Cái giếng bằng G, cái lánbằng H và hầm chứa đồ bằng N . Ta nối ngôi nhà đầu tiên với cái giếng G, cáilán H và hầm chứa đồ N , kéo dài các đường đi này theo các đường đi tới ngôinhà thứ hai. Ta thu được ba đường nối giữa hai ngôi nhà đó.

72

Page 77: SÁU PHƯƠNG PHÁP GIẢI CÁC BÀI TOÁN PHỔ THÔNG

Chương 6. Phương pháp sơ đồ

A B

G

H

N

I

II

III

Hình 6.5

Các đường đó chia mặt phẳng thành ba miền: I, II, III. Ngôi nhà thứ ba màta bỏ qua, nằm ở một chỗ nào đó của một trong các miền ấy

• Nếu ngôi nhà đó nằm trong miền I thì nó sẽ ở ngoài đường cong đóng kínbao quanh lán H.

• Nếu ngôi nhà đó thuộc miền II thì nó sẽ nằm ngoài miền III. Khi đó nóbị bao quanh bởi đường cong đóng kín mà cái hầm chứa đồ nằm ở ngoài.

• Nếu ngôi nhà đó thuộc miền III thì nó sẽ nằm ngoài miền I. Khi đó nó bịbao quanh bởi đường cong đóng kín mà cái giếng nằm ở ngoài.

Trong trường hợp thứ nhất không có đường đi từ ngôi nhà tới cái lán H. Vìkhông thuộc miền II, nên không thể có đường từ ngôi nhà thứ ba tới lán H màkhông cắt đường từ ngôi nhà A hoặc ngôi nhà B tới lán H.

Trong trường hợp thứ hai không có đường đi tới hầm chứa đồ N . Vì khôngthuộc miền III, nên không thể có đường từ ngôi nhà thứ ba tới hầm chứa đồN , mà không cắt đường từ A hoặc từ B tới hầm chứa đồ N .

Trong trường hợp thứ ba không có đường đi tới giếng G. Vì nằm ngoài miềnI, nên tất cả các đường đi tới giếng G đều hoặc cắt đường đi tới giếng G xuấtphát từ ngôi nhà A hoặc ngôi nhà B.

73

Page 78: SÁU PHƯƠNG PHÁP GIẢI CÁC BÀI TOÁN PHỔ THÔNG

Chương 6. Phương pháp sơ đồ

Vậy không thể có đường đi từ ba ngôi nhà A,B,C đến giếng, đến lán, đếnhầm chứa đồ, mà không cắt nhau.

Như vậy ta được điều phải chứng minh. �

Ví dụ 6.2.6. Trong một lớp mọi học sinh nam đều tham gia vào những nhómsở thích: Bóng đá, đá cầu, cầu lông. Qua tìm hiểu thấy rằng có 7 em tham giabóng đá, 6 em tham gia đá cầu, 5 em tham gia cầu lông, trong đó có: Một emtham gia cả ba môn, hai em tham gia vừa đá cầu vừa bóng đá, ba em tham giavừa cầu lông vừa bóng đá, bốn em tham gia vừa đá cầu vừa bóng đá.

Hãy xác định số học sinh nam của lớp?

Ta vẽ ba vòng tròn giao nhau, mỗi vòng tròn biểu thị một nhóm sở thích:Bóng đá (B), cầu lông (C), đá cầu (D).

Có một em tham gia cả ba nhóm, ta điền 1 vào phần chung của cả ba vòngtròn. Có hai em tham gia vừa đá cầu và cầu lông nhưng đã có 1 em tham giacả ba nhóm, vậy chỉ có 1 em tham gia đúng 2 nhóm sở thích vừa nêu. Ta điền1 vào phần chung của vòng tròn C và vòng tròn D (nằm ngoài phần chung củaba vòng tròn).

Lập luận tương tự, ta có 3 em tham gia đúng hai sở thích bóng đá và đá cầu,2 em tham gia đúng hai sở thích bóng đá và cầu lông, chỉ có một em tham giacầu lông. Ta điền các số này vào phần tương tự như hình vẽ. Từ đó, ta xác địnhđược số các em nam của lớp là 10.

1

1

1

1 2

3 1

C

B

D

Hình 6.6

Ví dụ 6.2.7. Năm người bạn là Nam, Thiện, Liêm, Khương, Bình có nghề

74

Page 79: SÁU PHƯƠNG PHÁP GIẢI CÁC BÀI TOÁN PHỔ THÔNG

Chương 6. Phương pháp sơ đồ

nghiệp là họa sĩ, thợ may, thợ mộc, đưa thư và cắt tóc. Họ sống trong cùng mộtthành phố nên có điều kiện gặp nhau thường xuyên.

Nam và Khương hay cùng đến hiệu may nơi người thợ may làm việc. Thiệnvà Bình sống cùng khu tập thể với người đưa thư. Liên vừa đóng vai trò chủ hôncho đám cưới của Thiện lấy con gái người thợ cắt tóc. Nam với Thiện chủ nhậtthường đi chơi cờ với người họa sĩ và người thợ mộc. Khương và Bình tối thứbảy hay đến chơi nhà người thợ cắt tóc. Người đưa thư thích nhất là tự cắt tóccho mình. Bình và Khương chưa bao giờ cầm bút vẽ. Hãy xác định nghề nghiệpcủa mỗi người?

Lời giải: Bài toán gồm có hai nhóm đối tượng:

• Nhóm thứ nhất gồm các bạn: Nam, Thiện, Liêm, Khương và Bình.

• Nhóm thứ hai gồm các nghề: Họa sĩ, thợ may, thợ mộc, đưa thư và cắt tóc.

Lấy một nhóm 5 điểm ghi tên 5 bạn có tên trên bằng các chữ cái đầu củatên, nhóm đối diện ghi tên 5 nghề mà họ làm kí hiệu như sau: 1: họa sĩ, 2: thợmay, 3: thợ mộc, 4: đưa thư, 5: cắt tóc.

Điểm tên người và điểm tên nghề được nối bằng nét liền, nếu người đó làmđúng nghề có tên được nối, trường hợp ngược lại được nối bằng nét đứt.

1) Xác định các đường nét đứt.Đầu tiên đưa vào các điều kiện về quan hệ đã cho để xác định các đường nét

đứt. Sau đó dựa vào các đường nét đứt để suy ra các đường nét liền. Cuối cùngdựa vào đó để suy ra đáp án.

- Theo bài ra ta có Nam và Khương hay cùng đến hiệu may nơi người thợmay làm việc suy ra Nam và Khương không làm thợ may, nên hai cặp điểm(Nam, thợ may) và (Khương, thợ may) được nối bằng nét đứt.

- Thiện và Bình sống cùng tập thể với người đưa thư, chứng tỏ Thiện và Bìnhkhông làm nghề đưa thư, nên các cặp điểm (Thiện, đưa thư) và (Bình, đưa thư)cũng được nối bằng đoạn nét đứt.

- Liên vừa làm chủ hôn cho đám cưới của Thiện lấy con gái người cắt tóc.Chứng tỏ Liên và Thiện không làm nghề cắt tóc, nên các cặp điểm (Liên, cắttóc) và (Thiện, cắt tóc) được nối bằng nét đứt.

- Nam với Thiện chủ nhật thường đi chơi cờ với người họa sĩ và người thợmộc, chứng tỏ Nam va Thiện không làm họa sĩ cũng không làm thợ mộc. Do đócác cặp điểm (Nam, họa sĩ), (Nam, thợ mộc), (Thiện, họa sĩ), (Thiện, thợ mộc),đều được nối bằng nét đứt.

75

Page 80: SÁU PHƯƠNG PHÁP GIẢI CÁC BÀI TOÁN PHỔ THÔNG

Chương 6. Phương pháp sơ đồ

- Bình và Khương chưa bao giờ cầm bút vẽ, chứng tỏ Bình và Khương đềukhông là họa sĩ. Bởi vậy, các cặp điểm (Bình, họa sĩ), (Khương, họa sĩ) đều đượcnối bằng nét đứt.

Khi đó ta có sơ đồ sau:

N

T

L

K

B

1

2

3

4

5

2) Xác định các đường nét liền.Dựa vào sơ đồ gồm các đường nét đứt để suy ra các đường nét liền.Vì mỗi người làm ít nhất một nghề và mỗi nghề có ít nhất một người làm,

nên mỗi điểm đều phải xuất phát ít nhất một đường nét liền, tức xuất phát tốiđa bốn đường nét đứt, nên dựa vào sơ đồ các đường nét đứt ta suy ra các đườngnét liền:

Điểm 1 đã có bốn đường nét đứt nối với các điểm N, T,K,B, nên đường nétliền của điểm này phải nối với điểm L.

Điểm 5 đã có bốn đường nét đứt nối với các điểm T, L,K,B, nên đường nétliền của điểm này phải nối với điểm N .

Điểm T đã có bốn đường nét đứt nối với các điểm 1, 3, 4, 5, nên đường nétliền của điểm này phải nối với điểm 2.

Điểm B chưa có đường nét đứt nối với các điểm 2, 3, nhưng điểm 2 đã cóđường nét liền nối với điểm T , nên đường nét liền của điểm B phải được nối với

76

Page 81: SÁU PHƯƠNG PHÁP GIẢI CÁC BÀI TOÁN PHỔ THÔNG

Chương 6. Phương pháp sơ đồ

điểm 3.Điểm 4 không có đường nét đứt nối ới các điểm N,L,K, nhưng điểm N đã

có đường nét liền nối với điểm 5, điểm L đã có đường nét liền nối với điểm 1,nên đường nét liền của điểm 4 phải nối với điểm K.

Vậy ta đã có sơ đồ hoàn chỉnh các đường nét liền. Dựa vào sơ đồ này ta suyra đáp án: Thiện làm nghề thợ may, Nam làm nghề cắt tóc, Liêm làm họa sĩ,Bình làm nghề thợ mộc và Khương làm nghề đưa thư.

77

Page 82: SÁU PHƯƠNG PHÁP GIẢI CÁC BÀI TOÁN PHỔ THÔNG

Kết luận

Luận văn đã nghiên cứu về sáu phương pháp phổ biến nhất để giải các bàitoán phổ thông. Mỗi phương pháp đều trình bày tóm tắt cơ sở lý thuyết và vậndụng các phương pháp đó vào giải một số bài toán trong chương trình trung họcphổ thông.

Khi biên soạn luận văn, tác giả đã cố gắng bám sát vào những dạng đề thihọc sinh giỏi. Hy vọng luận văn có thể là một tập tài liệu tham khảo có ích chohọc sinh và giáo viên các trường trung học phổ thông.

78

Page 83: SÁU PHƯƠNG PHÁP GIẢI CÁC BÀI TOÁN PHỔ THÔNG

Tài liệu tham khảo

[1] Bộ Giáo Dục và Đào Tạo - Hội Toán học Việt Nam (1997), Tuyển tập 30năm tạp chí Toán học và tuổi trẻ, NXB Giáo Dục.

[2] Bộ Giáo Dục và Đào Tạo (2013), Hình Học 11, NXB Giáo Dục Việt Nam.

[3] Nguyễn Văn Mậu (2007), Toán rời rạc và một số vấn đề liên quan.

[4] Nguyễn Văn Mậu, Trần Nam Dũng, Đặng Huy Ruận, Vũ Đình Hòa, ĐặngHùng Thắng (2007), Chuyên đề chọn lọc tổ hợp và toán rời rạc, NXB GiáoDục.

[5] Nguyễn Văn Nho (1989 - 2002), Olympic Toán học Châu Á Thái BìnhDương, NXB Giáo Dục.

[6] Phạm Minh Phương và nhóm giáo viên chuyên toán Đại học sư phạm HàNôi (2006), Các chuyên đề số học bồi dưỡng học sinh giỏi trung học cơ sở,NXB Giáo Dục.

[7] Nguyễn Mạnh Trinh (1983), Tuyển tập các bài thi vô địch Toán, NXB GiáoDục.

[8] Đặng Huy Ruận (2002), Bẩy phương pháp giải các bài toán logic, NXB Khoahọc và kĩ thuật.

79